DAT: Gen Chem Tests

Pataasin ang iyong marka sa homework at exams ngayon gamit ang Quizwiz!

16. Acetic acid (Ka = 1.8 x 10^-5) is titrated with KOH. Which pH indicator should you use to signal the end point of the titration? a. thymol blue; pH= 2-2.5 b. methyl yellow; pH= 3-4 c. bromocresol purple; pH= 5-7 d. cresol red; pH= 7.5-9 e. alizarine yellow; pH= 10.5-12

d. cresol red; pH= 7.5-9 -For the titration of a weak acid with a strong base, the equivalence point will be a weak base. For the pH indicator to properly signal the equivalence point, the indicator's pH range must be similar to the pH at the equivalence point. Only cresol red's pH indicator is in the weak base range. -Alizarine Yellow is used to identify strong bases, not weak

17. With increasing temperatures, which of the following occurs? a. entropy decreases b. total energy decreases c. enthalpy is constant d. entropy increases e. activation energy decreases

d. entropy increases -As the temperature increases, the disorder among the particles increases as solids begin to melt and liquids begin to vaporize [C]-enthalpy changes with temperature/isn't constant [E]-The activation energy is inherent to the reactants not to the temperature of the reaction

equation used to calculate percent change

% change= [(new amount-original amount)/original amount] x 100

9. 1 mol of HNO3 is dissolved in 54.0 grams of H2O. What is the mole fraction of HNO3?

0.25 mol fraction HNO3 = (mols HNO3) / (total mols of all components) mols HNO3 = 1 mol mols H2O = (54 g) / (18 g/mol) = 3 mol mol fraction HNO3 = 1 mol HNO3 / (1 mol HNO3 + 3 mol H2O) = 1/4 = 0.25

2. A chemist adds 26.5g of ammonium chloride to 10g of sodium hydroxide, which follows the reaction below. Assuming the reaction goes to completion, how much of which reactant is left in excess? NH4Cl + NaOH → NH3 + H2O + NaCl

0.25 mols, ammonium chloride 1. moles of ammonium chloride? -MW= 53 2. there are 0.50 mol NH4Cl (26.5g/53) 3. NaOH has a MW= 40 4. 10g/40= 0.25 mol NaOH 5. 0.25 mol NH4Cl reacts with 0.25 mol NaOH to form 0.25 mol NH3 6. 0.5 mol NH4Cl- 0.25 mol NH4Cl= 0.25 mol NH4Cl in excess

9. 10g of an unknown compound are added to water to form a 7.89 molar solution. If 2 liters of solution are present, what is the molar mass of the unknown compound?

10/[7.89*2] Molarity: M= mols solute/L solution 7.89= mols solute/2 mols solute= 7.89*2 -the question asks for molar mass which is g/mol -Dividing the grams given in the question by the moles just calculated would give the unknown compound's molar mass: MM = 10g / (7.89)(2)

5. Hydrogen peroxide (H2O2, 34 g/mol) decomposes into water vapor and oxygen gas. How many liters of water vapor are produced from the decomposition of 17.0 g of H2O2 at STP?

11.2 2 H2O2 --> 2 H2O + O2 17g H2O2 x (1 mol/34g) x (2 H2O/2 H2O2) = 0.5 mol H2O -remember, 1 mol of gas is equal to 22.4L at STP -if we have half a mole, then we will have 11.2L of gas at STP

22. When the following half-reaction is balanced in acidic medium, how many hydrogens are added to the left hand side of the half reaction after balancing? Cr2O72-(aq) → Cr3+(aq)

14 H+ 1. balance all atoms other than O and H: Cr2O7^2-(aq) → Cr^3+(aq) Cr2O7^2-(aq) → 2 Cr^3+(aq) 2. balance O by adding H2O: Cr2O7^2-(aq) → 2 Cr^3+(aq) + 7 H2O 3. balance H by adding H+: Cr2O7^2-(aq) + 14 H+ → 2 Cr^3+(aq) + 7 H2O

14. 2 A(g) + B(s) ⇌ 3 C(g) The above reaction is at equilibrium at STP and the molar concentrations found are [A] = 2.0 M, B = 2.0 mol, and [C] = 4.0 M. What is the equilibrium constant for this reaction at STP?

16 Ksp= [C]^3/[A]^2 = [4]^3/[2]^2 = 64/4 = 16 -solids and liquids are not included in equilibrium constant equations

24. A Lewis structure for O3 is: O = O - O. Including this structure, the total number of ground state resonance structures for this molecule is:

2

3. When the following equation is balanced using the smallest whole numbers, what is the coefficient in front of H2(g) on the product side of the reaction? Al(s) + HCl(aq) → AlCl3(aq) + H2(g)

3 2 Al(s) + 6 HCl(aq) → 2 AlCl3(aq) + 3 H2(g)

1. In a 3.21g sample of the hydrate, CuSO4 • 10H2O (339.8 g/mol), how many grams of water are expected?

3.21 x (180/339.8) % H2O= mass of H2O/mass of compound 1. total grams of H2O in the compound= (18g/mol)(10 mol)= 180g 2. sum of atomic weights of all the elements in the compound= CuSO4 • 10H2O = (63.54 + 32.06 + 4(16.00) + 180) = 339.8 3. % H2O= 180/339.8 4. mass of H2O in sample= 3.21g x (180.2/339.8)

1. Naturally occurring boron (10.8 u) exists in two isotope forms: B-10 (10.013 u) and B-11 (11.093 u). Which of the following statements is true?

B-11 is the most abundant isotope -Of the two isotopes, B-11 is the most abundant, since the atomic weight (average of the mass of the two isotopes) of boron is closest to the mass of B-11

1. Combustion analysis of a sample compound containing only C and H determines there was 18 g of C and 1 g of H in the sample. What is the empirical formula of this compound?

C3H2 mols C = (18g/1) x (1 mol/12g) = 1.5 mol C mols H = (1g/1) x (1 mol/1g) = 1 mol H -We have 1.5 mol C and 1 mol H. Chemical formulas cannot contain decimals, so we multiply by 2 to make our answer C3H2

8. Distillation analysis was performed on carbon tetrachloride (CCl4) and carbon tetrabromide (CBr4). Which molecule would be expected to have a higher boiling temperature and why?

CBr4, as CBr4 has a greater molar mass

15. What is the equilibrium constant expression for the following reaction? 2A(s) + 4B(aq) ⇌ 3C(aq) + D(l)

Kc = [C]^3 / [B]^4

2. Which of the following would be the empirical formula of a compound that has 16 moles of oxygen and 56 moles of nitrogen?

N7O2 -empirical formula= lowest mole number for each atom N: 56/8= 7 O: 16/8= 2

4. 15 moles of a N2 gas sample are placed in a 30L container at 33°C, what is the pressure (in atm) exerted by this gas sample?

P= [15*0.0821*306]/30 PV=nRT

27. The decrease in atomic radius from left to right across a period in the periodic table can be best explained by a. an increase in the number of protons b. an increase in atomic mass c. an increase in the number of inner shell electrons d. an increase in the number of neutrons e. a decrease in atomic mass

a. an increase in the number of protons

4. In which instance is a gas most likely to behave ideally?

at high temperatures and low pressures, because the molecules are far apart and not likely to interact

25. The first, second, and third ionization energy of an atom of a period 3 element is 738 kJ/mol, 1451 kJ/mol, and 7733 kJ/mol, respectively. The element is most likely a. sodium b. magnesium c. aluminum d. sulfur e. phosphorus

b. magnesium -magnesium contains 2 valence electrons -removing 2 valence electrons would move Mg to a stable octet (feels like noble gas) -removing a 3rd electron would remove Mg from a stable octet and therefore the 3rd ionization energy would be much higher (1451 kJ/mol --> 7733 kJ/mol)

28. What type of nuclear reaction is involved when C-11 decays to B-11? a. alpha emission b. positron emission c. beta emission d. neutron swap e. gamma emission

b. positron emission 11 11 6 C → 5 B -Note that the mass number stays the same but the atomic number (number of protons) decreases. This is what occurs in positron emission (as well as electron capture, though electron capture is not an option here)

28. Which particles are commonly referred to as nucleons? a. electrons and protons b. protons and neutrons c. neutrons and electrons d. beta particles and positrons e. alpha particles and beta particles

b. protons and neutrons -The nucleon are the particles that make up the nucleus. The nucleus is comprised of neutrons and protons -Alpha particles and beta particles are comprised of nucleons, but are not referred to as nucleons themselves

12. If the conjugate base of a molecule has a pKb of 1.4, what would you expect the molecule to be? a. strongly acidic b. weakly acidic c. neutral d. weakly basic e. strongly basic

b. weakly acidic -lower numbers in pKb correlate with stronger bases, and higher numbers in pKb correlate with stronger acids -strong acids have a weak conjugate base -strong bases have a weak conjugate acid -the conjugate base given in this problem is strongly basic which means that the molecule must be weakly acidic

18. The ∆Hf for Br(g) is +193 kJ/mol. What is the bond dissociation energy of a Br-Br bond? a. +193 kJ/mol b. -193 kJ/mol c. +386 kJ/mol d. -386 kJ/mol e. +96.5 kJ/mol

c. +386 kJ/mol -Bond dissociation energies are always positive because we have to put in energy to break the bond -The ∆Hf for Br(g) is: (1/2) Br2(l) -> Br(g) ; ∆Hf = +193 kJ/mol -double this reaction, we find: Br2(l) → 2Br(g) ; ∆Hf = (2)(+193) = +386 kJ/mol

22. Which of the following compounds has the same sulfur oxidation number as SO2? a. SO3 b. S2O3 c. SO3^2- d. SO4^2- e. HSO4^-

c. SO3^2- -oxygen's oxidation number= -2 -if the oxygens each have an oxidation number of -2, and the overall compound is neutral, the sulfur must have an oxidation number of +4 SO3^2- S + 3(-2) = -2 S = +4

12. Which of the following best describes a compound that reacts as both an acid and a base? a. nonpolar b. neutral c. amphoteric d. metallic e. charged

c. amphoteric

30. To separate a homogeneous mixture of salt water, which of the following pieces of laboratory equipment is necessary? a. buret b. pipet c. condenser d. filter e. separatory funnel

c. condenser -Homogeneous mixtures are often separated using distillation. In distillation, the different boiling points of the substances are used to separate the mixture. The mixture is boiled, until one of the compounds evaporates. The evaporated compound is then immediately cooled through the condenser and collected on the other end in a flask.

10. The solvation of ionic compounds in water is due to which of the following? a. reactions with H3O+ and OH- b. diffusion c. dipoles of water d. high specific heat of water e. cohesive and adhesive forces

c. dipoles of water -the oxygen takes on a slightly negative charge and the hydrogens take on a slightly positive charge. These charges help to attract the cation and anions in an ionic compound and help to dissolve ionic compounds.

8. Pure elemental gases, such as O2 and N2, have very low melting points, at around -220°C. What is one reason that this is the case? a. pure gases are in gas form and thus are never solid b. pure gases are hard to freeze c. pure gases are purely covalently bonded and are non polar d. pure gases have many strong intermolecular forces e. pure gases are unreactive

c. pure gases are purely covalently bonded and are non polar -The melting point of a substance is due to the strength of intermolecular forces between the molecules. Pure elemental gases are covalently bonding and are non-polar. As a result, they have very minimal intermolecular forces acting on the molecules. This results in a weak bonding structure in the solid, and thus a low melting point

9. Which of the following solutions has the lowest freezing point? a. 0.20 m C6H12O6 b. 0.20 m Ba(NO3)2 c. 0.20 m H2SO4 d. 0.20 m Na3PO4 e. 0.20 m K2CrO4

d. 0.20 m Na3PO4 van't Hoff factor [D] will break down into 4 solute particles in solution (3 Na+ and 1 PO4-3) -the largest van't Hoff factor depresses the freezing point the most, giving us the lowest freezing point

25. In the Lewis dot diagram for SF4, how many total electrons, shared and unshared, are surrounding the central element? a. 4 b. 6 c. 8 d. 10 e. 12

d. 10 -sulfur and phosphorus can have as many as 12 electrons in its valence shell b/c it is in the third period and has s, p, & d unfilled orbitals -sulfur has 6 valence electrons normally -in SF4, each bond to sulfur from fluorine contains two electrons, so 4 bonds x 2 electrons = 8 electrons, plus the two electrons from the lone pair = 10 electrons total

11. What is the mass percent of methanol in a solution prepared by diluting 16 grams of methanol with 32 grams of water? a. 8% b. 16% c. 25% d. 33% e. 50%

d. 33% mass % = grams solute/grams solution mass %= 16/(16+32)= 33%

23. In an electrolytic cell: a. the reaction qt the anode is reduction b. the reaction at the cathode is oxidation c. the flow of electrons goes from cathode to anode d. the cathode is the negative electrode e. the reaction is spontaneous

d. the cathode is the negative electrode -oxidation always occurs at the anode and reduction always occurs at the cathode -in a galvanic cell the anode is negative and the cathode is positive -flow of electrons always goes from anode to cathode -the reaction is non-spontaneous in electrolytic cells and spontaneous in galvanic cells

25. All of the following are nonpolar molecules EXCEPT one. Which one is the EXCEPTION? a. XeF4 b. CH4 c. CO2 d. PF5 e. SO2

e. SO2

6. Which of the following liquids would have the highest volatility? a. a liquid with hydrogen bonds b. a liquid with ionic bonds c. a liquid with a high viscosity d. a metal in liquid form e. a liquid with London dispersion forces

e. a liquid with London dispersion forces -Volatility is the ability of a liquid to evaporate. Liquids that readily evaporate have weak intermolecular attractions and can turn into gas at low temperatures

14. Which of the following acids has the strongest conjugate base? a. pKa = 1.66 b. pKa = 3.35 c. pKa = 4.74 d. pKa = 7.52 e. pKa = 10.6

e. pKa = 10.6 -the stronger the acid, the weaker its conjugate base is -the higher the pKa of an acid, the weaker the acid is

20. A first order reaction has a half-life of 90 minutes. What is the rate constant in hour-1?

(0.693)/1.5 The half-life for a first order reaction is: t1/2 = 0.693/k We are given time in minutes and they want the final rate constant in hours. There are 1.5 hours in 90 minutes. Now, simply substitute in the values given, and we find that k = (0.693)/1.5

4. A fixed sample of an ideal gas occupies a volume of 1.2 L at 30°C and 1.0 atm. What volume will the gas occupy if the temperature is increased to 60°C at constant pressure?

(1.2)(333)/ 303 (V1/T1) = (V2/T2) (1.2 L/303 K) = (V2)/333 K V2 = (1.2 L*333 K)/303 K

1. A volumetric flask weighs 185g when empty and 380g when filled with "liquid A" (density of 2 g•mL-1). If the flask is filled with 160g of "liquid B", what is the density of "liquid B" in g•mL-1?

(2)(160)/(380-185) density= P(MM)/RT= m/v

10. What is the final volume (in mL) when 300mL of 0.8 M KCl is concentrated to 0.9 M KCl?

(300)(0.8/0.9) M1V1 = M2V2 (0.8)(300) = (0.9)V2

22. Calculate the standard cell potential of a voltaic cell based on the following half-reactions. Ni^2+ + 2 e- --> Ni E(reduction) = -0.28 V Sn^2+ + 2 e- --> Sn E(reduction) = -0.136 V

+0.144 V -the half-reaction that occurs at the cathode is the one with the more positive reduction potential -although both reduction potentials given are negative, Sn is less negative than the reduction potential of Ni. Therefore, Sn is the cathode (reduction), and Ni is the anode (oxidation) standard cell potential of a voltaic cell is: Ecell = E(red) + E(oxide) Ecell = -0.136 + 0.28 = 0.144 V

24. A standard voltaic cell is constructed using Cu metal in 1.0 M Cu(NO3)2(aq) and an unknown metal in a 1.0 M solution of its nitrate salt. The cell voltage is 0.47 V when the Cu half-cell is the cathode. What is the standard reduction potential of the unknown metal? EoCu2+/Cu = 0.34 V

-0.13 V The potential of a voltaic cell is equal to the reduction potential of the half reaction at the cathode (reduction) plus the oxidation potential of the half-reaction at the anode (oxidation) Ecell = E(red) + E(oxid) 0.47 V = 0.34 V + E(oxid) E(oxid) = 0.13 V convert from oxidation to reduction potential by inverting the sign E(red) = -0.13 V

17. H2O + CO2 → H2CO3 Two steps are needed to form the above reaction. If the first step must be reversed and halved and the second step must be doubled, which of the following would be the correct mathematical representation?

-1/2( ΔHstep1) + 2( ΔHstep2) Since the first step of the reaction must be reversed, the enthalpy of step 1 must be multiplied by -1. Likewise if the step also has to be reduced in half, then the enthalpy of step 1 has to be multiplied by 1/2. The second step had to be doubled, so the enthalpy of the second reaction had to be multiplied by 2.

19. A given substance has a boiling point of 177oC, and a molar heat of condensation of -90 kJ/mol. What is the molar entropy of condensation of the substance?

-200 J/mol K The temperature at which a liquid boils is the same as the temperature at which condensation occurs. At the condensation point, the gaseous and liquid phases are in equilibrium with each other, and at equilibrium Gibbs free energy (ΔG) = 0 ΔG = ΔH - TΔS 0 = ΔH - TΔS ΔS = ΔH condensation/ T boiling ΔH condensation = -90 kJ/mol = -90000 J/mol T boiling = 177 + 273 = 450 K ΔS condensation = -90000/450 = -9000/45 = -200 J/mol K

17. The standard enthalpy of formation of liquid methanol is provided below, C(graphite) + 2H2(g) + 1⁄2O(g)2 → CH3(l)OH ΔHf = -239 kJ Which of the following would most likely be the value of the standard enthalpy of formation of gaseous methanol?

-200 kJ ΔHf for the gas is more positive than the ΔHf we were provided for the liquid -A reaction occurs, and the bond breakage and formation has a net release of energy, which is why ΔHf is negative. -to go from a liquid to a gas requires an input of energy -we would expect a gas to have a more positive (less negative) ΔH than a liquid

5. Calculate the work done by an ideal gas on its surroundings when the gas is heated at constant pressure of 1 atm from volume 50L to 100L at 298K.

-50 L*atm w = -P∆V = -P(V2 - V1) w = -(1 atm)(100 L - 50 L) w = -50 When the system does work on the surroundings, we have negative work of the system, and energy will be lost.

15. Consider the following 1.3 M aqueous solution of acetic acid: HC2H3O2 (aq) -> H+ (aq) + C2H3O2 (aq), Ka = 1.8 x 10^-5 What would be the pH of the solution at equilibrium?

-log[√(1.3)(1.8x10^-5)] [H+] = √(Ka)(M) pH = -log[H+]

10. A student dissolved 10.1 g of KNO3 (molecular mass of KNO3 = 101.10 g/mol) into 1000 mL of water. He then took 10 mL of the resulting solution and diluted it to 100 mL. What is the molarity of the second solution?

0.010 M mol KNO3 = 10.1 g/ 101.10 mol KNO3 = 0.10 mol concentration of KNO3 = mol KNO3/volume in liters [KNO3] = 0.10 mol/ 1.0 L [KNO3] = 0.10 M M1V1=M2V2 M1= 0.10 M KNO3 V1= 10 mL = 0.01 L V2= 100 mL = 0.1 L (0.10M KNO3)(0.01L) = M2(0.1L) 0.001 mol = M2(0.1 L) M2 = 0.001 mol/0.1 L M2 = 0.010 M

16. At 250oC, the Keq of H2 + I2 ⇌ 2HI is equal to 100. If at equilibrium the concentration of HI is equal to 0.4M and the concentration of I2 is equal to 0.1M, then what is the concentration of H2 at equilibrium?

0.016 M Keq = [HI]^2 / [H2][I2] 100 = [0.4]^2 / [H2][0.1] [H2]*10 = 0.16 [H2] = 0.16/10 = 0.016 M

28. Strontium-83 decays by beta emissions and has a half-life of 32 hours. How much of a 0.88 mg sample of Sr-83 remains after 4 days?

0.11 mg amount remaining = original amount * (1/2)^t t = number of half lives 4 days = 4 x 24 hrs = 96 hrs 96/ 3 = 32 t = 3 half lives amount remaining = 0.88 mg * (1/2)^3 after one half-life: 0.88 x 0.5 = 0.44 mg after the 2nd half-life: 0.44 x 0.5 = 0.22 mg after the third half-life: 0.22 x 0.5 = 0.11 mg remaining

11. 100 mL of 0.50 M CaCl2(aq) is mixed with 100 mL of 0.25 M H2SO4(aq). What is the final concentration of [Ca2+] in this solution? CaSO4 is insoluble in solution.

0.125 M Ca^2+ + SO4^-2 → CaSO4 (0.100 L)(0.25 M) = 0.025 mol SO4^2- (0.100 L)(0.50 M) = 0.050 mol Ca^2+ 0.050 mol Ca^2+ - 0.025 mol SO4^2- = 0.025 mol Ca^2+ remaining free in the solution 0.025 mol/ (100mL + 100 mL)= 0.025 mol/0.200 L= 0.125 M [Ca^2+]

21. Consider the following reduction half-reactions: Ni^2+ (s) + 2e- → Ni (s) E°= -0.28 Cl2 (g) + 2e- → 2 Cl^- (aq) E°= 1.36 Which of the following is the Eº cell for the reduction of chlorine?

0.28 + 1.36 E° cell= E° cell reduction + E° cell oxidation -The half-reaction for nickel has a negative E° indicating that the reaction prefers to go in the reverse direction. Therefore, the nickel reduction half-reaction must be flipped to yield an oxidation reaction (+0.28) -The half-reaction of the chlorine is positive, indicating that chlorine prefers the forward reaction and wants to be reduced

2. The following reaction was performed in water: H2SO4 + 2KOH → K2SO4 + 2H2O How many moles of K2SO4 are produced from 2.5 L of 0.3M KOH and 1 L of 0.3M H2SO4?

0.30 mol 2.5 L * 0.3 M = 0.75 mol KOH 1 L * 0.3M = 0.3 mol of H2SO4 2 moles of KOH are consumed for each mole of H2SO4 0.75 mol / 2 KOH = 0.375 mol 0.3 mol / 1 H2SO4 = 0.3 mol there is 1 mol of H2SO4 for every mol of K2SO4 0.3 mol H2SO4 * (1 mol K2SO4/ 1 mol H2SO4) = 0.3 mol K2SO4

15. A 1.0 L sample of an aqueous solution contains 0.20 mol of NaCl and 0.10 mol of MgCl2. What is the minimum amount of moles of AgNO3 that must be added to the solution to precipitate all of the Cl- ions as AgCl(s)?

0.40 mol -assume that all of the Cl- dissociates in NaCl and MgCl2. Since NaCl only has one chlorine atom, when it dissociates it will produce 0.20 mol of Cl-. Since MgCl2 has 2 chlorines, it will produce 0.20 mol of Cl- (0.10 mol x 2 mol of Cl-). In total, we will have 0.40 mol of Cl- in the solution. Since there is only one silver atom in AgNO3, we will need 0.40 mol of Ag to react with all 0.40 mol of Cl-

4. A mixture of gas is composed of 0.5 mol Ar, 1.5 mol He, and 2.0 mol of N2. What is the partial pressure of N2 in the mixture if the total pressure is 0.95 atm?

0.48 atm mol fraction N2= (mols N2)/(total mols) mol fraction N2= 2.0/(0.5+1.5+2.0)= 2.0/4.0= 0.5 PN2= (mol fraction N2)(Ptotal) PN2= 0.5*0.95 atm= 0.48 atm

5. CuO(s) + CO2(g) <--> CuCO3(s) + heat At a certain temperature, Kp = 2.0. What is the partial pressure of CO2 (g) in the above reaction?

0.50 atm Kp= [products]/[reactants] = [1]/ [PCO2] -only aqueous and gaseous compounds are included in the K values. Therefore, the partial pressure of CO2 is the only compound that is affecting the Kp. 2.0 atm = 1/PCO2 PCO2= 1/ 2.0 atm PCO2= 0.5 atm

13. What is the concentration of [CN-] in a 0.2 M solution of HCN? The Ka of HCN is 5 × 10^-10

1 x 10^-5 [H+]= [CN-]= √(Ka)(M) [CN-]= √(5x10^-10)(2x10^-1) [CN-]= √10x10^-11 [CN-]= √1x10^-10 10^-10(1/2) 10^-10/2 [CN-]= 1 x 10^-5

1. Hydrogen gas and nitrogen gas react to produce ammonia. How many liters of ammonia can be produced from 2 liters of hydrogen gas and 2 liters of nitrogen gas at STP?

1.3 L -The described reaction is the Haber process: N2 + 3 H2 → 2 NH3 We can treat the liters of gas the same as mols in this case. This is because at a constant temperature and pressure, the volume of the gas is directly proportional to the number of mols. 2L H2 x (2 NH3/3 H2)= 4/3= 1.33 mol NH3 2L N2 x (2 NH3/1 N2)= 4 mol NH3

16. At equilibrium, what is the concentration of Mn^2+ in a solution that is buffered at pH = 12, if the Ksp = 1.6 × 10^-13 for Mn(OH)2?

1.6 x 10^-9 -In a buffered solution, the concentration of OH- will be resistant to change, and therefore will be related to the pOH pOH= 14 - 12 = 2 [OH^-] = 10^-pOH = 10^-2 Mn(OH)s (s) --> Mn^2+ (aq) + 2 OH^- (aq) Ksp = [Mn^2+][OH^-]^2 1.6 x 10^-13 = [Mn^2+]*(10^-2)^2 1.6 x 10^-13 = [Mn^2+]* 10^-4 [Mn^2+] = (1.6 x 10^-13)/ 10^-4 [Mn^2+] = 1.6 x 10^-9 M

1. Nitric oxide, NO, rapidly reacts with O2 to form NO2. How many grams of NO are required to completely react with 1.75 g of O2?

1.75*(1/32)*(2/1)*(30/1) 2 NO + O2 → 2 NO2 -We convert from grams of O2 to mols of O2 -Convert from mols of O2 to mols of NO -Convert mols of NO to grams of NO

2. How many O atoms are present in 1.0 g of Al2O3 (101.96 g/mol)?

1.8 x 10^22 mol Al2O3 = (1 g Al2O3)*(1 mol Al2O3/ 101.96 g) = ~ 0.01 mol number of Al2O3 units ~ (0.01 mol Al2O3)* (6.02 x 10^23) = ~6.02 x 10^21 since there are 3 O atoms per Al2O3 # of O atoms = 3* (6.02 x 10^21) = 18 x 10^21 = 1.8 x 10^22

10. What is the molar solubility of BaF2 (Ksp = 1.8×10-7) in 0.10 M sodium fluoride?

1.8×10^-5 M BaF2(s)--> Ba^2+(aq) + 2F^-(aq) Let x = molar solubility of BaF2 Ksp = [Ba^2+][F^-]^2 Ksp = 1.8×10^-7 = (x)(0.10)^2 x = (1.8×10^-7)/(1.0×10^-2) x = 1.8×10^-5 M

3. An element X forms an oxide X2O3 with a mass ratio of 0.45 g X / 1 g O. What is the atomic mass of X?

10.8 amu mass of X = (mass of O)*(1 mol O/ MW of O)* (2 mol X/ 3 mol O)* (MM of X) Substituting 0.45 g for the mass of X and 1 g for the mass of O gives: 0.45 g X = 1 g O * (1 / 16 g/mol)* (2/3)* (MM of X) 0.45 = 2 / (3*16)* (MM of X) 0.45 = 1/24 * (MM of X) 0.45 * 24 = MM of X MM of X = 10.8 amu

20. After 2.2 billion years, how much of a 100-gram sample of Uranium-235 would remain, assuming a half-life of 704 million years?

100*[1/2]^(2200/704) Amount remaining = original amount x (1/2)t the half-life of Uranium-235 is 704 million years. If it has been 2.2 billion years which is equivalent to 2,200 million years, then the number of half lives would be: 2,200/704 Substituting the original amount of 100 into the half-life equation results in: 100(1/2)^(2200/704)

11. What is the boiling point of a solution of 5.0 mol of glucose dissolved in 1000 mL water? (Kb for water = 0.512 °C / m; assume density of water = 1g/mL)

102.6 °C ΔTb = Kb*m*i Kb = 0.512 °C m = 5.0 mol i = 1 (glucose is 1 "thing") boiling point of pure water = 100°C -Rounding off 0.512 to 0.5 gives: 0.5 * 5.0 = 2.5 2.5 + 100 °C = 102.5 °C

12. A 1.0 M solution of (CH3)2NH reacting with water, according to the reaction shown below, has a Kb equal to 4 x 10^-6. What is the pH of the solution at 25°C? (CH3)2NH (aq) + H2O (l) ⇌ (CH3)2NH2^+ (aq) + OH^- (aq)

11.3 Kb = [(Ch3)2NH2^+][OH^-] / [(Ch3)2NH2] Kb = 4 x 10^-6 = x^2 / 1.0 x = √4 x 10^-6 x = 2 x 10^-3 x = [OH] = 2 x 10^-3 pOH = -log[OH] pOH = -log(2 x 10^-3) pOH = ~2.7 pH = 14 - pOH pH = 14 - 2.7 pH = 11.3

11. What volume of 0.5M H2SO4 is needed to neutralize 250mL of 0.5M KOH?

125mL Neutralization Formula: n1*M1*V1= n2*M2*V2 -H2SO4 has two moles of H+ ions (n1=2), while OH only has one mole of OH-(n2=1) 2*0.5*V1= 1*0.5*250 V1= 125mL

12. If the Ka of acetic acid is 1.8 × 10-5, what is the pKb of acetate?

14 + log (1.8 x 10^-5) (Ka)(Kb) = Kw = 10^-14 pKa + pKb = 14 pKb = 14 - pKa pKb = 14 - (-log(Ka)) pKb = 14 - (-log(1.8 × 10^-5)) pKb = 14 + log (1.8 × 10^-5)

22. When the following reaction is balanced in acidic solution using whole number coefficients, what is the coefficient of H+? CrO42- → Cr2O72-

2 1. balance chromium: 2 CrO4^2- → Cr2O7^2- 2. balance oxygens by adding H2O: 2 CrO4^2- → Cr2O7^2- + H2O 3. balance the hydrogens: 2H^+ + 2CrO4^2- → Cr2O7^2- + H2O 4. add e- to balance the charges: -not needed in this case

23. The valence electron in ground state lithium can be described by which set of quantum numbers?

2, 0, 0, +1/2 -The electron configuration of ground state lithium is (1s^2)(2s^1) -(2s^1) describes the valence electron specifically, where n= 2 -[s=0, p=1, d=2, f=3] the electron is in the s orbital, so l= 0 -ml [+1, 0, -1] refers to each of the 3 P orbitals -since lithium has no p orbitals, ml= 0

2. What are the coefficients that must be used to balance the combustion of butane: C4H10 + O2 → CO2 + H2O

2, 13, 8, 10

11. A student wants to prepare a 100 mL 0.500 M solution of NaOH from solid NaOH. How much NaOH (in grams) would the student need to weigh?

2.0 g figure out the number of moles of NaOH we need, and from there we can use molar mass to determine the mass g NaOH = M*V*molar mass g NaOH = (0.5 mol/L)(0.100 L)(40 g/mol) g NaOH = (0.05 mol)(40 g/mol) = 2.0 g

28. Uranium-235 is a radioactive nuclide that decays by alpha emission. What is the product of decay of Uranium-235?

231 90 Th Radioactive decay by alpha emission involves production of a He-4 nucleus -therefore the product must have a mass number of 235 - 4 = 231 and an atomic number of 92 - 2 = 90

29. An experiment was performed with -25% error. If the theoretical yield of the experiment is 3.45 grams, how much of the product was never collected and measured?

3.45 - [3.45 - 0.25(3.45)] % error = experiment value - theoretical value/ theoretical -0.25 = (x - 3.45) / 3.45 cross multiply: -0.25(3.45) = x - 3.45, add 3.45 over to the other side to isolate x x = 3.45 - [0.25(3.45)] the amount that was not collected= the difference between the theoretical amount and the experimental amount: 3.45 - [3.45 - 0.25(3.45)]

3. The pressure of a gas is increased from 100 mmHg to 120 mmHg. If the initial temperature of the gas was 300K, which of the following could be the final temperature of the gas?

360K -Pressure and temperature are directly related, so if the pressure increased, the temperature must also increase. The pressure increased by about 20%, so you would expect the temperature to do the same, increasing to somewhere around 360K.

15. What is the molar solubility of PbSO4 in 4.0 × 10-2 M Pb(NO3)2 solution? The Ksp for PbSO4 is 1.6 × 10-8

4.0 x 10^-7 M

2. Ammonia is formed according to the reaction below. A chemist mixes 21 grams of nitrogen gas and 18 grams of hydrogen gas in a 2.0 L vessel. How many grams of hydrogen gas will be consumed? N2(g) + 3H2(g) → 2NH3(g)

4.5 grams 1. calculate how many mols of each Molar mass of N2 = 28 g/mol Molar mass of H2 = 2 g/mol Moles N2 = 21 g/28 = ¾ mol = 0.75 mol Moles H2 = 18 g/2 = 9 mol 1 N2 is consumed for every 3 H2. Therefore, consuming 0.75 moles of N2 would require 2.25 moles of H2 (1:3 ratio). And since we have 9 moles of H2, H2 is in excess and N2 is the limiting reagent. -If 2.25 moles of H2 are consumed, and H2 has a molar mass of 2 g/mol, 4.5 grams of hydrogen will be consumed in this reaction.

2. A sample of hydrated magnesium sulfate weighs 21.0 g: MgSO4 • xH2O It is placed inside of an oven until all of the water is vaporized, the anhydrous form weighs 12.0 g. What is the value of x in the hydrated salt? (MgSO4 MW = 120 g•mol-1.)

5 "x"= mol H2O/mol MgSO4 g H2O= 21 g - 12 g= 9 g mol H2O= 9 g/ 18 g*mol= 0.5 mol MgSO4= 12 g/120 g*mol = 0.10 "x"= 0.5/0.1 = 5 MgSO4 • 5 H2O

21. How many electrons are transferred in this half-reaction? Mn2+ → MnO4-

5 Mn reactant phase: Mn^2+ Mn product phase: x + (-2)(4)= -1 x= +7 the difference between Mn^2+ and Mn^7+ is 5 electrons

10. How much water should be added to 10 mL of 3.00 M HCl(aq) in order to dilute it to a 2.00 M solution of HCl(aq)?

5 mL M1V1 = M2V2 M1 = 3.00 M HCl(aq) V1 = 10 mL M2 = 2.00 M HCl(aq) (3.00)(10) = (2.00)(V2) V2 = 15 mL 15 - 10 = 5 mL -The question asks how much water should be ADDED to the original solution, so we subtract by the original amount, V1.

20. A first order reaction has an initial concentration of 40M and a half- life of 2 hours. After 6 hours how much of the initial concentration is left?

5.0 M concentration = original concentration 6 hours is 3 half-lives ½ * ½ * ½ = 1/8 the original amount 1/8 of 40 M is 5.0 M

21. For the reaction 2NO(g) + O2(g) → 2NO2(g), the concentration of NO was monitored from 0 to 100 seconds. Given the following data, what is the average rate of disappearance of NO from 0 to 100 seconds? t = seconds [NO] = M t = 0; [NO] = 1.0 x 10^-3 t = 100; [NO] = 5.0 x 10^-4

5.0 x 10^-6 M/s average rate of disappearance = -Δ[A] / Δt = - ([A]t2 - [A]t1 / t2 - t2) rate of NO = -[(5.0 x 10^-4 M)- (1.0 x 10^-3 M)] / (100 s - 0 s) rate of NO = -[(5.0 x 10^-4 M)- (10 x 10^-4 M)] / (100 s - 0 s) rate of NO = -(-5.0 x 10^-4) / 100 s the negative cancel out: rate of NO = 5.0 x 10^-4 / 1.0 x 10^2 (10^-4)-(10^2) = 10^-6 rate of NO = 5.0 x 10^-6

2. A 5.0m saturated solution of cyclohexanol (MM = 100 g/mol) dissolved in water has a density of 15 g/mL. What is the molarity of this solution?

50 molar Mass = moles * molar mass Mass = molality * kg of solvent * molar mass Mass = (5.0 mol/kg) * 1kg * 100 g/mol Mass = 5.0 moles * 100 g/mol = 500 grams And we know we have 1 kilogram (1000 grams) of water for every 5 mols of cyclohexanol. Therefore, Total mass of solution = 500 grams cyclohexanol + 1000 grams of water Total mass of solution = 1500 grams Density = mass / volume 15 g/mL = 1500 g / volume Volume = 1500 g / 15 g/mL Volume = 100 mL = 0.1 L M = mol / L solution M = 5 mol / 0.1 L M = 50 mol / L

5. What is the temperature at which 64 g of O2 will occupy a volume of 8.2 L at 760 Torr?

50.0 K PV = nRT n = mol O2 = (64 g)/(32 g/mol) = 2 mol P = 760 Torr = 1 atm V = 8.2 L R = 0.0821

13. What is the OH-(aq) concentration in 9.0 M CH3NH2? The Kb for CH3NH2 is 4 × 10^-4

6 x 10^-2 [OH-]= √(Kb)(M) = √(4x10^-4)(9.0) = √36x10^-4 = √36 x √10^-4 = 6 x 10^-2

1. How many grams of oxygen are needed to completely react with 0.50 moles of C3H8 (58 g/mol)?

80 g C3H8 + 5 O2 → 4 H2O + 3 CO2 g oxygen = (0.5 mol C3H8) x (5 mol O2/ 1 mol O2) x (32 g O2/ 1 mol O2) = 80 g of oxygen

16. A small amount of aqueous NH3 is added to a solution of Cu^2+ and a precipitate forms. However, when excess aqueous NH3 is added, the former precipitate dissolves. Which statement best explains why the precipitate dissolves with excess aqueous NH3?

Cu^2+ forms a complex ion with NH3 -Complex ions are those such as Cu(NH3)4^2+. The water molecules that normally surround a cation are replaced with some other electron pair donor (NH3 in this case) Reaction with dilute NH3: Cu^2+ + 2 NH3 + 2 H2O → Cu(OH)2 + 2 NH4^+ Reaction with excess NH3: Cu^2+ + 4 NH3 → Cu(NH3)4^2+

24. Which of the following compounds would have Lewis dot diagrams constructed with the same number of valence electrons as NH4+?

H2O -NH4+ has 5 valence electrons from the nitrogen, 4 valence electrons from the hydrogens and one valence electron is subtracted from the "+" charge (5 + 4 - 1) = 8 valence electrons -H2O has 2 valence e- from hydrogen and 6 from oxygen (2 + 6) = 8

13. Based on the reaction below, which of the following statements is correct? HClO3 + H2O ⇌ H3O^+ + ClO3^-

H2O acts as a base and H3O^+ is its conjugate acid

27. Place the following elements in order of DECREASING ionization energy: Si, He, F, Cs

He > F > Si > Cs -ionization energy increases as you move to the right and up on the periodic table

6. A gas filled balloon contains triptane gas (C7H16 ; MM = 100 g/mol) and another unknown gas. This balloon develops a pinhole leak. The unknown gas effuses out of the balloon five times as fast as triptane gas. Which of the following could be the unknown gas?

Helium gas, He (4 g/mol) Graham's law of effusion tells us rateA/rateB = √(MM B)/(MM A) -We know that gas x effuses five times as fast as triptane, so rateX/rateC7H16 = √(MM C7H16)/(MM X) 5 = √(100)/(MM X) 25 = 100 / MM X MM X = 100/25 = 4 g/mol

21. In a given electrochemical cell, a spontaneous reaction occurs where metal A forms the cathode and metal B forms the anode. Based on the information provided, which of the following statements is/are correct? I. Oxidation occurs at the anode II. The reduction potential of metal A > the reduction potential of metal B III. This reaction represents an electrolytic cell

I and II only -A spontaneously reacting electrochemical cell is known as a voltaic or galvanic cell -A non-spontaneous reacting electrochemical cell is known as an electrolytic cell, and requires the input of energy

15. A researcher is examining data provided to her regarding two solutions. The solubility (mol/L) of solid AgCl in water was found to be 1.34 x 10-5 and the solubility (mol/L) of solid AgBr in water was 7.07 x 10-7. Which of the following conclusions can be made?

Ksp for AgCl is greater than Ksp for AgBr -the higher the molar solubility is, the larger the Ksp value will be. AgCl, with a molar solubility of 1.34 x 10^-5 would have the higher Ksp

14. The following acid-base reactions have K > 1. Which of the following choices correctly rank the acids in increasing strength? HF + CN- → HCN + F- NH3 + HF → NH4+ + F- HCN + NH3 → NH4+ + CN-

NH4^+ < HCN < HF We can deduce the relative strength of the acids from these reactions, where the stronger acid is always on the reactant side, -Reaction I: HF is stronger than HCN -Reaction II: HF is stronger than NH4+ -Reaction III: HCN is stronger than NH4+

16. The ΔH for which of the following reactions is the standard enthalpy of formation (ΔH°f) for sodium chloride?

Na (s) + 1/2 Cl2 (g) --> NaCl (s) -The ΔH°f for a compound is the enthalpy change for the formation of one mole of the compound from reactants in their most stable forms at standard conditions -Sodium is solid and chlorine is a gas at standard conditions. To form one mole of the compound we must use 1 mole of Na (s) and 1/2 a mole of Cl2 (g).

11. Which of the following is the correct formula of the conjugate base of HSO41-?

SO4^2-

chemical kinetics graphs: which of the following reactions will occur the fastest?

The reaction with the lowest activation energy (difference in energy between the reactant and the transition state or the highest point along the reaction path) will occur the fastest.

30. A student measured a liquid reagent using a graduated cylinder. Which of the following is a proper way of reporting the volume of the liquid in the graduated cylinder?

The value of each line on this graduated cylinder is 0.1 mL. Between each division, we can read (estimate) up to 0.01 mL. To read the volume of the liquid, we look at the level of the lower meniscus and in this case, it is between 5.6 and 5.7 mL. We then estimate the last digit, and the proper reading is 5.67 mL. The last digit is an estimate, but the reading should be reported up to the nearest 0.01 mL

16. What is the concentration of Pb^2+ at equilibrium if the Ksp of PbCl2 equals 1.8 x 10^-5 in an aqueous solution of pure water?

[(1.8 x 10^-5) / (4)]^1/3 PbCl2(s) ⇌ Pb^2+(aq) + 2 Cl^-(aq) Ksp = [Pb^2+][Cl^-]^2 Ksp = [x][2x]^2 Ksp = 4x^3 1.8 x 10^-5 = 4x^3 x = [(1.8 x 10^-5) / (4)]^1/3

8. Sulfuric acid is stored in a protective container to prevent it from corroding the container. If 250mL of the 4.5M stock solution of sulfuric acid is to be transferred to an empty flask, what volume of water in liters must be added to the flask to reduce the sulfuric acid's molarity to 2.2M?

[(4.5*250)/2.2] - 0.250 Dilution: M1V1=M2V2

22. ZnCl2 is electrolyzed with a current of 7.00 amperes for 3 minutes. Which of the following is equal to the maximum amount of Zn(s) that is produced in grams? (1 faraday = 96,500 coulombs)

[(7)(180)(65)]/ (2)(96,500)] grams product = [(amps)(seconds)(MW)] / (n)(F) mols product = (amps)(seconds) / (n)(F) -(amps) = amperes, or the current -(seconds) = the amount of time the reaction took place in seconds -(MW) = molecular weight of the product -(n) = moles of electrons in the reaction -(F) = Faraday constant

17. 10 kJ of heat was applied to a 150 g sample of copper at 50°C. The specific heat of copper is 0.39 J·g-1·°C-1. What is the final temperature of the copper after the heat is applied in °C?

[10,000/(150)(0.39)] + 50 q = mCpΔT q = heat m = mass Cp = specific heat ΔT= (Tf - Ti) = change in temperature

3. If compound X3P2 has a molar mass of 135 g/mol, what is the atomic weight of the unknown element X?

[135- 2P]/3 -3X + 2P = 135, so to solve for the atomic weight of unknown element X 3X = 135 - 2P X = (135 - 2P) / 3

20. Calculate the heat of enthalpy, ΔH for the following reaction, based on the bond energies provided: CH4(g) + 4Cl2(g) → 4HCl(g) + CCl4(g) Bond energies: C-H = 413 kJ / mol Cl-Cl = 243 kJ / mol H-Cl = 432 kJ / mol C-Cl = 330 kJ /mol

[4(413) + 4(243)] - [4(432) + 4(330)] ΔH = [bonds of reactants] - [bonds of products] reactants: CH4 has 4 C-H bonds 4Cl2 has 4 Cl-Cl bonds products: 4HCl has 4 H-Cl bonds CCl4 has 4 C-Cl bonds

24. Which of the following is the ground state electron configuration for Cr^2+?

[Ar] (4s^0)(3d^4) -the ground state electron configuration for normal Cr is [Ar] (4s^1)(3d^5) -when an atom becomes ionized, we remove electrons from the highest level (n value) first -Therefore, Cr will lose 1 electron from the 4s orbital before it loses 1 electron from the 3d orbital

24. Rank the following in order of increasing neutron/proton ratio. a. 24Mg b. 207Pb c. 65Zn

a < c < b -The number of protons of an element is equal to its atomic number. The number of neutrons of an element is equal to the mass number minus the number of protons. The superscript in the symbol indicates the mass number of the element [24Mg]= 24-12 = 12; 12/12 = 1 [207Pb]= 207-82 = 125; 125/82 = ~1.5 [65Zn]= 65-30 = 35; 35/30 = ~1.2

26. Rank the following in order of decreasing first ionization energy (highest first ionization energy to lowest first ionization energy) a. N b. Li c. O

a > c > b -First ionization energy tends to increase across a period (with some exceptions) and decrease down a group -Li is to the left of the other 2 elements and will have the smallest ionization energy -Hund's rule states that every orbital in a subshell is singly occupied with one electron before any one orbital is doubly occupied. So only in oxygen will we have electrons pair up in one of the 2p orbitals -it will be easier to remove one of the electrons from the paired electron orbital in oxygen making oxygen's ionization energy lower compared to nitrogen

10. Find the osmotic pressure (in atm) of a 0.50 M aqueous solution of NaCl at 27 degrees Celsius. a. (2)(0.50)(0.082)(300)

a. (2)(0.50)(0.082)(300) osmotic pressure: pi= iMRT -van't Hoff factor (i)= NaCl--> 1Na+ + 1Cl- = 2 -M= 0.50 -R= 0.082 -T= 27+273= 300K

11. Which of the following contains the greatest quantity of moles of Cl- ions? a. 200 mL 0.30 M CaCl2 b. 100 mL 0.10 M AlCl3 c. 300 mL 0.1 M NaCl d. 50 mL 0.4 M MgCl2 e. 150 mL 1.0 M AgCl

a. 200 mL 0.30 M CaCl2 (0.200 L)*(0.30 M)*(2 mol Cl^-) = 0.12 mol

26. What is the difference between the bond angles of water and methane? a. 5 degrees b. 10.5 degrees c. 61.5 degrees d. 70.5 degrees e. they have the same bond angle

a. 5 degrees -methane [CH4] is sp3 tetrahedral with a 109.5 degree angle -water [H2O] is sp3 bent with a 104.5 degree angle

22. Which of the following acts as a reducing agent? a. Cl^- b. IO3^- c. F2 d. Zn^2+ e. Cl2

a. Cl^- -A reducing agent is oxidized/will give up electrons to become more positive -an oxidizing agent is reduced/receives electrons to become more negative [B]-IO3^- looks like a possibility, but the oxidation state on iodine is +5, and it isn't likely to give up more electrons

25. All of the following atoms are isoelectronic EXCEPT one. Which one is the EXCEPTION? a. F^- b. S^2- c. K^+ d. Ar e. Ca^2+

a. F^- all other atoms have the same number of total electrons We have Ar or [Ne](3s^2)(3p^6) as our electron configuration

3. Which of the following compounds contains the highest mass of oxygen per gram of compound? a. H2O (18 g/mol) b. CO2 (44 g/mol) c. NO2 (46 g/mol) d. CaC2O4 (128 g/mol) e. KMnO4 (158 g/mol)

a. H2O (18 g/mol) # oxygen / molar mass [H2O]= 1/18 [CO2]= 2/44 = 1/22 [NO2]= 2/46 = 1/23 [CaC2O4]= 4/128 = 1/32 [KMnO4] = 4/158 = ~1/40

26. Aqueous ammonium sulfide is added to a solution of iron (II) chloride. What are the spectator ions in this reaction? I. NH4+ II. S2- III. Cl- IV. Fe2+ a. I and III b. I and II c. II and IV d. III and IV e. II and III

a. I and III -Generally, salts with ammonium will dissolve, and salts with chloride, bromide, or iodide will also dissolve -salts with nitrate ions (NO3-) are soluble -iron sulfide (the product) is NOT soluble and will precipitate as a solid NH4+ (aq) + S2- (aq) + Fe2+ (aq) + Cl- (aq) → FeS (s) + NH4+ (aq) + Cl- (aq) -the ions that are the same on both sides are spectator ions and get canceled out

14. Consider the following reaction: MgCO3(s) → MgO(s) + CO2(g), which of the following would be the correct equilibrium constant expression Keq? a. Keq = [CO2] b. Keq = [MgO] c. Keq = [MgO][CO2]

a. Keq = [CO2] -When writing equilibrium constant expressions, you must divide the concentration of the products by the concentration of the reactants -pure solids and liquids are not included in these expressions

27. Which of the following electron configurations represents the atom with the largest atomic radius? a. [Kr] (5s^1) b. [He] (2s^2)(2p^4) c. [Ar] (4s^2)(3d^10)(4p^4) d. [Kr] (5s^1)(4d^10) e. [Ar] (4s^1)(3d^10)

a. [Kr] (5s^1) -atomic radius increases as you go down and to the left on the periodic table [A] = Rb [B] = O [C] = Se [D] = Ag [E] = Cu

13. A Lewis acid can be described as a species that: a. can accept an electron pair b. can donate an electron pair c. can accept a proton d. can donate a proton e. increases the concentration of [H+]

a. can accept an electron pair -Those species that can accept an electron pair (ex. H+) are deemed acids, and those species that can donate an electron pair (ex. OH-) are bases. -[C] and [D] are Bronsted-Lowry definitions of acids and bases and revolve around protons. Molecules that can accept a proton are deemed bases, and molecules that can donate a proton are deemed acids.

14. Adding sodium oxalate to a solution of oxalic acid causes the pH to: a. increase due to the common ion effect b. decrease due to the common ion effect c. remain constant because the resulting solution is a buffer d. increase due to shifting the Ka e. decrease due to shifting the Ka

a. increase due to the common ion effect -Sodium oxalate is a base, and adding a base to anything will cause an increase in pH. This increase in pH will be due to the common ion effect in this case, because oxalic acid and oxalate share a common ion. -Adding reactant does not shift the Ka.

24. Which of the following best describes the bond character for hydrochloric acid? a. polar covalent b. covalent c. ionic d. hydrophobic e. hydrogen bonding

a. polar covalent -Ionic bonds are generally formed between elements from the far left and the far right of the periodic table. Hydrochloric acid (HCl) may appear ionic since it is made from hydrogen and chlorine which appear on opposite sides of the periodic table. However, in HCl, the hydrogen and chlorine share an electron to complete each other's valence shells. Therefore, the bond cannot be ionic. Only pure elements (N2, for example) are purely covalent. Since chlorine is much more electronegative than hydrogen, it will hog the electron more than the hydrogen, creating polarity, so therefore this is a polar covalent bond.

10. When NaCl is added to distilled water, each of the following properties would increase EXCEPT one. Which one is the EXCEPTION? a. vapor pressure b. molarity c. osmotic pressure d. molality e. boiling temperature

a. vapor pressure -with less solvent molecules at the surface, less solvent can escape the liquid and become a gas causing vapor pressure to decreases -as more solute is added to a solution, all of the following properties increase in value: 1. Molarity: (mol solute/ L solution) 2. Osmotic Pressure: (i*M*R*T) 3. Molality: (mol solute/ kg solvent) 4. Boiling temperature elevation: (i*Kb*m)

21. Buried steel propane tanks are often attached to a zinc plate to minimize corrosion. Which of the following explains why the zinc plate will corrode instead of the steel tank? a. zinc acts like an anode and is more easily oxidized than steel b. zinc is more easily reduced than steel c. steel acts like a cathode and is more easily oxidized than zinc d. steel acts like an anode and is more easily reduced than zinc

a. zinc acts like an anode and is more easily oxidized than steel -REDuction occurs at the CAThode/cathode gains mass or is protected -OXidation occurs at the ANode/anode loses mass -electrons will be oxidized from the zinc and the zinc will corrode instead of the steel tank

26. Which of the following combinations of elements would most likely have the smallest bond length?

an atom with high ionization energy and another atom with high electron affinity -high ionization energy= small atomic radii -high electron affinity= small atomic radii -low metallic character= small atomic radii -small atomic radii= shorter bond length

5. A dentist uses an ML-6 type cylinder, which has an internal volume of 165 liters. How many moles of nitrous oxide are in the tank (assume the apparatus is at sea level; the ambient temperature is 22° C)? b. (1)(165)/(0.082)(295)

b. (1)(165)/(0.082)(295) PV= nRT n=PV/RT P= 1 atm V= 165L R= 0.082 T= 22+273= 295K

4. A fixed sample of an ideal gas at 298K occupies a certain volume x at a certain pressure y. If the volume of the gas is increased to 2x, what will the pressure of the gas be in terms of y? a. (1/4)y b. (1/2)y c. y d. 2y e. 4y

b. (1/2)y -The pressure of a given sample of an ideal gas at constant temperature is inversely proportional to its volume P1V1 = P2V2 -Doubling the volume at constant temperature would decrease the pressure by half

26. Predict the bond order of the N-O bond in NO3^- a. 1.00 b. 1.33 c. 1.50 d. 2.00 e. 3.00

b. 1.33 The actual structure of NO3^- is the mixture or hybrid of three resonance structures. All three Lewis structures of NO3^- are equally dominant. The N-O bond in NO3^- has a bond order of 1.3 (average of the N-O bonds in the three structures). This is from the four bonds averaged out over the three positions. 4/3 = 1.33

2. In air bags, the reaction of sodium azide (NaN3, 65 g/mol) to form sodium and nitrogen gas is triggered by an electric current, thus expanding the air bag. If there are 65 grams of sodium azide in the air bag, how many mols of nitrogen gas are formed? a. 1 mol b. 1.5 mols c. 3 mols d. 5 mols e. 0.1 mols

b. 1.5 mols

12. An unknown solution is found to have a pOH of 4.12, which of the following would be equal to the [H+] of the solution? a. 10^-pOH b. 10^-(14-pOH) c. 10^(14-pH) d. 10^-(14-pH) e. 10^(14-pOH)

b. 10^-(14-pOH) pH + pOH = 14 [H+]= 10^-pH [H+]= 10^-(14-pOH)

3. How many grams of lead(II) sulfate (303 g/mol) are needed to react with sodium chromate (162 g/mol) in order to produce 0.162 kg of lead(II) chromate (323 g/mol)? a. 112.8 b. 151.5 c. 189.2 d. 289.6 e. 212.5

b. 151.5

19. Which of the following reactions will be spontaneous at all temperatures? a. 2H2 (g) + O2 (g) → 2H2O (g) ΔH = -483.6 kJ b. 2 NCl3 (g) → N2 (g) + 3 Cl2 (g) ΔH = -460.1 kJ d. 2Na (s) + Cl2 (g) → 2NaCl (s) ΔH = -821.8 kJ

b. 2 NCl3 (g) → N2 (g) + 3 Cl2 (g) ΔH = -460.1 kJ -A spontaneous process is one where ΔG < 0, or when ΔH < 0 and ΔS > 0 -when ΔH is positive, the reaction will never be spontaneous at all temperatures If Δngas > 0, ΔS > 0 [A]: Δngas = 2-3 = -1, thus ΔS<0; the reaction will be spontaneous at low temperature [D]: Δngas = 0-1 = -1, thus ΔS<0; the reaction will be spontaneous at low temperature [B]: Δngas = 4-2 = 2, thus ΔS>0; the reaction is spontaneous at all temperatures

21. The half-life of a radioactive isotope of strontium is 6 months. What mass of a 32 gram sample of this isotope will remain after one year? a. 16 g b. 8 g c. 4 g d. 2 g e. 1 g

b. 8 g -One year is two half-lives. The sample will fall to half its value two times. 32 g * (1/2) * (1/2) = 8 g.

3. What is the sum of the coefficients of the complete balanced equation from combusting C2H6O? a. 6 b. 9 c. 12 d. 18 e. 24

b. 9 Combustion reaction: hydrocarbon (CH) or alcohol (CHO) + O2 --> always produces CO2 + H2O 1 C2H6O + 3 O2 → 2 CO2 + 3 H2O 1+3+2+3= 9

9. The boiling point of a 1.0m aqueous solution is 101.5°C. The molal boiling point constant for water is 0.512°C•m-1. Which of the following substances is most likely dissolved in the water? a. NaCl b. Na2SO4 c. C6H6 d. CaCO3 e. K3 PO4

b. Na2SO4

13. A researcher has a solution of 3M nitric acid that she needs to neutralize. In order to do this, she decides to add the acid to a solution to neutralize it. Which of the following would neutralize the acid the fastest? a. Br- b. Na3PO4 c. NH3 d. Cl- e. H2O

b. Na3PO4 -Na3PO4 is able to absorb 3 protons, because it dissociates into the very basic PO4^3- ion -the other answer choices would only absorb 1 proton

15. The following exothermic reaction was allowed to reach equilibrium: P4(s) + 10Cl2(g) ⇌ 4PCl5(g) + heat Which of the following changes would increase the amount of PCl5(g)? a. increasing the volume of the container b. decreasing the temperature c. adding more P4(s) d. adding a nonreactive gas such as He e. adding a catalyst

b. decreasing the temperature -decreasing the temperature of an exothermic reaction causes a shift towards the product side [A]-increasing the volume of the container would decrease the pressure and since there are more gas molecules on the reactant side, the reaction would shift left towards the reactants [C]-solids are not included in the equilibrium expression [D]-He is an inert gas and isn't involved in the reaction [E]-adding a catalyst doesn't affect equilibrium

7. Which process would be used to separate two miscible liquids? a. filtration b. distillation c. decantation d. disproportionation e. crystallization

b. distillation -Filtering or straining the homogeneous mixture would not help to separate the two liquids. -Decantation is the process to remove the top layer of liquid after the precipitate has settled at the bottom. Decantation would not work as a precipitate is not formed since the two liquids are miscible

7. All of the following statements accurately describe why mercury forms a convex meniscus in a glass thermometer EXCEPT one. Which one is the EXCEPTION? a. the molecular forces of cohesion exceed the forces of adhesion b. mercury tends ti become ionized and the net charge repels the glass c. nonpolar and polar molecules tend to repel each other d. the forces between mercury molecules are stronger than the forces between the mercury and glass e. capillary action pulls the edges of the mercury down

b. mercury tends ti become ionized and the net charge repels the glass

7. A student is examining a crystalline solid. Which of the following findings would be most expected during analysis of this substance? a. short range order of atoms b. sharp, well defined melting temperature c. malleable shape deformation d. random lattice structure e. irregular breakage patterns

b. sharp, well defined melting temperature -Crystalline solids have long range order. There is a consistent, crystal lattice network of atoms that is well defined, and repeated throughout the structure. Crystalline solids have sharp, well defined, characteristic melting temperatures, and break with a cleavage along a very straight plane -Amorphous solids have only short range order. There is no consistent structure of atoms throughout the solid. This leads to a broad range of melting temperatures for any given amorphous solid, and they have irregular breakage patterns.

30. Which of these elements will have a vigorous reaction in the presence of pure oxygen? a. boron b. sodium c. germanium d. sulfur e. neon

b. sodium -Sodium and the group 1A metals will react violently with pure oxygen. The reactions are normally combustions -Group 2A metals also react with oxygen, but not to the extent of the 1A elements -Germanium, sulfur, and boron react with oxygen to make oxides

27. An extracted pure compound appears silvery white, exhibits high conductivity to heat and electricity, yet has a very low melting point. Which of the following elements is a possible match for this compound? a. platinum b. sodium c. germanium d. selenium e. iodine

b. sodium -alkali metals, the first group in the periodic table, display metallic properties due to their low first ionization energies, but they have low melting points due to their weak inter-atomic bonding [A]- platinum is a metal, but it has a high melting point [C]-germanium is a metalloid and has a high melting point/is a semi-conductor [D]-selenium is a nonmetal/not highly conductive [E]-iodine does not conduct electricity

15. Consider the following reaction: AgCl(s) ⇌ Ag^+1(aq) + Cl^-1(aq) Which of the following would occur if NaCl were added to the mixture of AgCl? a. the amount of Ag^+1 will increase b. the amount of AgCl will increase c. the amount of Na+ will decrease d. the amount of NaCl will increase e. the amount of AgCl will decrease

b. the amount of AgCl will increase -if NaCl is added to the mixture, it will dissolve to form Na^+1 and Cl^-1 ions in the solution. NaCl and AgCl share a common ion: Cl^-1 -the addition of NaCl will shift the reaction to the left, since there is more Cl^-1 in the solution therefore reducing the solubility of the AgCl. This will increase the amount of AgCl in the solution.

28. Element X has a nucleus that is stable against decomposition. Which of the following must be true of Element X? a. the effective nuclear charge is high b. the nuclear binding energy is high c. the ionization energy is high d. the electron shielding is high e. the bond dissociation energy is high

b. the nuclear binding energy is high [A]-the effective nuclear charge is the net positive charge experienced by an electron in a multi-electron atom. It explains the atomic radii trend across periods. [D]-the electron shielding effect dictates the atomic radii trend down a group. [E]-bond dissociation energy deals with compounds, not nuclei.

15. Consider the below chemical equilibrium in a closed vessel at a constant temperature. The easiest method to measure the equilibrium constant for this system is to measure: Na2CO3(s) ⟷ CO2(g) + Na2O(s) a. the temperature of the reaction b. the pressure of the CO2 gas c. the molar concentrations of all the reactants d. the forward and reverse rate constants e. the mass of the solid present

b. the pressure of the CO2 gas -Equilibrium constants do not include solids or liquids, such as Na2CO3 and Na2O. So the equilibrium constant equation here would be Kp = [CO2] -The pressure of the CO2 gas is the only determinant

5. Consider two balloons of equal volume at the same temperature; one containing 1 mol of He and the other containing 2 mol of Ar. Which of the following statements is true? a. the two gases have the same pressure b. the two gases have the same average kinetic energy c. the two gases have the same average speed d. the two gases have the same entropy

b. the two gases have the same average kinetic energy [A]-pressure is directly proportional to the amount of the gas sample. There are more moles of Ar gas, and therefore it has a higher pressure [C]-the average speed of gases is dependent on both the temperature and the molar mass. [D]-there are more moles of Ar gas, and therefore it has a higher entropy.

23. The central atom of a molecule has 4 pairs of electrons, and the molecule is V-shaped (bent). Which of these statements is always true regarding this molecule? a. the central atom is sp^2 hybridized b. there are two bonding orbitals on the central atom c. the bond angle around the central atom is 120° d. the molecule is miscible with hexane e. the molecule can participate in hydrogen bonding

b. there are two bonding orbitals on the central atom -For a molecule with a central atom having 4 pairs of electrons to be V-shaped, the central atom must have 2 pairs of bonding electrons and 2 pairs of lone pairs

16. Ca^2+(aq) + SO4^2-(aq) → CaSO4(s) Which of the following must be true for the above reaction? a. ΔS > 0 b. ΔS < 0 c. ΔG < 0 d. ΔH > 0 e. ΔS = 0

b. ΔS < 0 -two aqueous solutions are coming together to form a solid precipitate. -The resulting product is more "organized" than the two aqueous reactants, therefore the reaction must have a negative change in entropy (-ΔS).

18. ATP hydrolysis is a spontaneous reaction, and occurs according to the equilibrium below. Based on this information, which of the following properties will have a positive value during this reaction? ATP4 + H2O ⇄ ADP3 + Pi + H2 + 49 kJ a. ΔH system b. ΔS universe c. ΔG system d. Δmass universe e. ΔT system

b. ΔS universe -During any spontaneous process, the entropy of the Universe increases [A]-heat is a product (exothermic); ΔHsystem will be negative [E]-an exothermic reaction releases heat, which causes the temperature of the system to decrease [C]-Gibbs free energy (ΔG) would be negative for this reaction since it is spontaneous [D]-The law of conservation of mass tells us that the matter (mass) in the Universe is constant, and cannot be created or destroyed, only change forms

12. The following mixtures (assume volumes are additive) will result to a buffer solution EXCEPT one. Which one is the EXCEPTION? a. 0.10 M CH3COOH + 0.10 M NaCH3COO b. 0.40 M HCN + 0.20 M KOH c. 0.050 M HCl + 0.075 M NaOH d. 0.125 M NH4Cl + 0.100 M NH3 e. 0.100 M HCl + 0.20 M NH3

c. 0.050 M HCl + 0.075 M NaOH -there is no source of a weak acid/conjugate base or a weak base/conjugate acid. HCl is a strong acid and NaOH is a strong base. Reaction of HCl and NaOH forms H2O and NaCl. -buffer solution consists of a weak acid and its conjugate base or a weak base and its conjugate acid [A]= buffer solution: weak acid + its conjugate base [B]= buffer solution: HCN will react with KOH in a 1:1 molar ratio to form H2O and KCN (which is the conjugate base of HCN) [D]= buffer solution: weak base (NH3) + its conjugate acid (NH4^+) [E]= buffer solution: excess weak base (NH3) + its conjugate acid

29. A student measures the mass of a compound to be 9.01 g and its volume to be 11.0 mL. What is the density in g/mL? a. 0.8 b. 0.82 c. 0.819 d. 0.8191 e. 0.81909

c. 0.819 -We have 3 significant figures in 9.01g and 3 significant figures in 11.0mL, so our answer must have 3 significant figures

28. Which compound(s) listed below would result in a basic solution when placed in water? 1. BaO 2. CO2 3. Na2O 4. P4O10 a. 1 only b. 2 only c. 1 and 3 d. 2 and 4 e. 4 only

c. 1 and 3 -Metal oxides produce basic solutions in water. -Non-metal oxides produce acidic solutions in water. -transition metal oxides can be either acidic or basic depending on the oxidation number of the metal. The higher the oxidation number, the more acidic the corresponding oxide. For example, CrO (Cr has a +2 charge) is a basic oxide, but CrO3 (where Cr has a +6 charge) is an acidic oxide.

24. If the bond length of H2 is experimentally found to be 74pm and the bond length of Br2 is experimentally found to be 228pm, what would be the bond length of H-Br? a. 138 pm b. 142 pm c. 151 pm d. 156 pm e. 159 pm

c. 151 pm H2= 74 pm H= 74/2= 37 pm Br2= 228 pm Br= 228/2= 114 pm H-Br= 37 + 114 = 151 pm

6. The bond length of the chlorine molecule, Cl2, is 199 pm. On the other hand, a pure carbon structure, such as diamond, has C-C bond lengths of 154 pm. What is the bond length of a C-Cl bond? a. 154 pm b. 218 pm c. 176 pm d. 120 pm e. 188 pm

c. 176 pm C-Cl = [154+199]/2= 176.5

4. An unknown gas has a density of 1.79 g•L-1 at 273K and 1 atm. Identify the gas. a. He b. Ne c. Ar d. Kr e. Xe

c. Ar density= P(MM)/RT= m/v -All you need to know is that there is 1 mol in 22.4L at STP, which is 273K (0° Celsius) and 1 atm. 1.79g/L x 22.4L/1mol= 40g/mol --> Ar

1. The empirical formula of a compound is found to be CO3. What is the molecular formula if the molar mass of the compound is 180 g/mol? a. CO9 b. C2O6 c. C3O9 d. C4O12 e. C6O18

c. C3O9 CO3 = 60 g/mol 180 / 60 = 3x multiplier CO3 × 3 = C3O9

1. A sample of chromium oxide is 76.5% chromium by weight. What is the simplest formula of the oxide? a. CrO3 b. CrO2 c. CrO d. Cr2O e. Cr2O3

c. CrO [mass of chromium/mass of sample] * 100 = % chromium 52+16= 68 [52/68]* 100= 76.5%

19. The rate law of a certain reaction is rate = k[X][Y]^2. The units of k is(are): a. s^-1 b. M^-1s^-2 c. M^-2s^-1 d. M^-1 e. M^-1s^-1

c. M^-2s^-1 -add up the exponents in the rate law. We see the [X] has 1 as its exponent, and [Y]2 has 2 as its exponent. 1+2 = 3, so this is a third-order reaction First order: s^-1 Second order: M^-1s^-1 Third order: M^-2s^-1

7. Sodium oxide, Na2O, has a melting point of 1132 °C. Which of the following molecules would be expected to have a melting point that is greater than that of sodium oxide? a. NaCl b. Cl2 c. MgO d. Ne e. CH4

c. MgO -Sodium oxide represents an ionic compound, which have very high melting points compared to nonionic compounds. We therefore can eliminate [B], [D], and [E] -melting point of ionic compounds increases with increasing charge and decreasing size according to Coulomb's Law Comparing MgO to Na2O - the radii of Mg and Na are quite similar (right beside each other in the periodic table), and MgO has greater charges (+2 and -2) compared toNa2O (+1 and -2) so MgO will have the higher melting point

29. Parallax error may occur when: a. an experiment doesn't account for change in altitude b. a measurement is not properly converted between the metric and customary systems c. a meniscus in a buret is not observed directly straight on d. an object under water is observed at an angle fro above the surface

c. a meniscus in a buret is not observed directly straight on -parallax error may occur when a pointer on a meter or surface of a liquid in a buret or pipette is not observed from directly straight on

26. All of the following elements exist as diatomic molecules EXCEPT one. Which one is the EXCEPTION? a. nitrogen b. oxygen c. helium d. fluorine e. hydrogen

c. helium -There are seven elements that exist as diatomic molecules: the halogens fluorine - F2 chlorine -Cl2 bromine -Br2, iodine - I2 nitrogen - N2 oxygen - O2 hydrogen - H2

7. Which of the following is a characteristic of network covalent solids? a. malleability b. conductivity c. high melting point d. high luster e. frail

c. high melting point - Luster, conductivity, and malleability are characteristics of a metal -A network covalent solid does not contain metals [it only contains non-metals]

22. Which of the following is true of reaction intermediates? a. reactions with intermediates always take longer to proceed than reactions without intermediates b. the step of the overall reaction that contains an intermediate is always the slow step c. intermediates never appear in rate equations d. intermediates never are consumed, they only acts as cats;ysts for the reaction e. intermediates appear transiently in all chemical reactions

c. intermediates never appear in rate equations -A reaction intermediate is a short lived (transient) molecule that forms in a multistep reaction. It is a different, separate existing molecule that is unique from the reactants and the products [E]-although all reactions involve bonds being broken and formed (transition states), not all reactions have reaction intermediates forming and being consumed (multistep reactions)

6. Water molecules in an aqueous solution will have the strongest interactions with ions with which of the following characteristics? a. small charge and small size b. small charge and large size c. large charge and small size d. large charge and large size

c. large charge and small size -A polar water molecule will be more likely to interact with an ion with more charge "like dissolves like" -think about the charged ion and water molecule as two magnets. As you bring magnets closer together, you begin to feel their force of attraction. We need to minimize the distance between the water molecule and charged ion by choosing an ion with a smaller size

27. Which of the following descriptions is correct regarding nitrogen and phosphorus? a. nitrogen has a larger atomic radius than phosphorus b. nitrogen is a metalloid and phosphorus is a nonmetal c. nitrogen can bond to fewer atoms than phosphorus d. nitrogen is less electronegative than phosphorus e. nitrogen less readily forms pi bonds than phosphorus

c. nitrogen can bond to fewer atoms than phosphorus -nitrogen and phosphorus are both nonmetallic elements -Nitrogen, being on top of phosphorus, is more electronegative and contains fewer electrons causing it to have a smaller atomic radius -Because of its smaller size, nitrogen can bond to fewer atoms than phosphorus -nitrogens smaller size also allows it to approach another nitrogen atom more closely and form π bonds more effectively

26. Which of the following elements reacts explosively with water? a. lead b. iodide c. potassium d. oxygen e. argon

c. potassium -All the group 1 elements, the alkali metals, are known to react very vigorously with water to produce hydrogen gas

5. All of the following are assumptions of an ideal gas EXCEPT one. Which one is the EXCEPTION? a. all gas molecules are in constant, rapid, and random motion b. there are no attractive or repulsive forces between the gases c. the average kinetic energy is proportional to the number of gas molecules d. the volume of the molecules is negligibly small compared to the volume of the gas e. all collisions within the gas are perfectly elastic

c. the average kinetic energy is proportional to the number of gas molecules -average kinetic energy only depends on the temperature of the system

19. Which of the following is true for the energy diagram of a reversible exothermic reaction?

c. the reverse reaction has a higher activation energy than the forward reaction -a larger activation energy means the reaction would proceed slower -the enthalpy of the reactants is larger than the products, which is why exothermic reactions have a negative ΔH

14. At 100 oC, the pH of a neutral solution is 6.1. Which of the following statements is true for this solution at this temperature? a. the pOH of this solution is 7.9 b. this solution at pH= 6.5 is acidic c. this solution with a pOH of 7 is acidic d. the pH + pOH of this solution is 14 e. at 50 oC, the neutral pH of this solution is 6.1

c. this solution with a pOH of 7 is acidic [A,D]-A neutral solution is one where [H+] = [OH-]. Thus, if the pH of a neutral solution is 6.1, then its pOH is also 6.1, and pH + pOH = 12.2 [B]-Since at 100 oC, the neutral pH is 6.1, then a solution with pH 6.5 is basic [E]- Kw = 1 x 10-14 at 25 oC, but the value changes with temperature. We would not expect the pH of this neutral solution to still be 6.1 when the temperature changes from 100 oC to 50 oC

18. During the analysis of an exothermic reaction process, mass increases from 15g to 35g, volume decreases by 1⁄3, work performed is 65 kJ, and the Gibbs free energy change is positive. Which of these changes represents a path function (non-state function)? a. enthalpy b. volume c. work d. mass e. Gibbs free energy

c. work -a path function (non-state function) is one in which the process matters. Path functions depend on the transition or change of the state -heat and work are path functions -enthalpy, volume, mass and Gibbs free energy are all state functions

18. Consider the following reaction at 298K: 2NH3 → 3H2 + N2 ∆H = -92,400 J/mol ∆S = 310 J/K Which of the following is true? a. ΔT > 0 b. ΔG = 0 c. ΔG < 0 d. ΔG > 0 e. ΔS < 0

c. ΔG < 0 ∆G = ∆H -T∆S ∆G = -92,400 - (298)(310)

17. Describe the reaction below. PCl3(g) + Cl2(g)→ PCl5(g) ; ΔHf = -87.9 kJ/mol a. spontaneous at all temperatures b. non spontaneous at all temperatures c. ΔG < 0 only at low temperatures d. spontaneous only at high temperatures e. ΔG < 0 only at high temperatures

c. ΔG<0 only at low temperatures -ΔH= a negative number (exothermic) -entropy change ΔS= negative (the products are more ordered than the reactants) because the reactants are 2 moles of gas and the product is only 1 mole of gas -low temperature = adding a small positive number will keep ΔG<0 -high temperature = adding a large positive number will change ΔG>0

6. Which of the following is an example of a compound that has polar bonds but a non-polar shape? a. SO2 b. H2O c. N2 d. CF4 e. NH3

d. CF4 1. Bond Polarity: -there is a large difference in the electronegativities of the elements that make up a bond 2. Molecular Polarity: -deals with the shape of the molecule, looking for symmetry -Symmetry leads to a non-polar molecule CF4 and N2 are the only options that have a symmetrical shape, but the N-N bond has the same electronegativity and is not polar

23. Which compound has a carbon atom with the highest oxidation state? a. CH3OH b. CHF3 c. CH2O d. CO2 e. Na2C2O4

d. CO2 Oxidation number of C + 2*oxidation number of O = 0 x + 2*(-2) = 0 x = +4

9. Two separate solutions are created. One using water and KCl, and the other using water and CaCl2. Assuming the molality is equal in both solutions, which of the following is true? a. KCl will have the lower freezing point b. KCl will have the higher boiling point c. CaCl2 will have the lower boiling point d. CaCl2 will have the higher boiling point

d. CaCl2 will have the higher boiling point and lower freezing point ∆Tb = m × Kb × I -the molality is the same for both solutions, this can be disregarded. Both compounds are ionic and since the solvent is water, the Kb is also the same for both equations, which for water is 0.512 -KCl: "I" = 2 ions -CaCl2: "I" = 3 ions

10. Which of the following 0.25 M solutions would exhibit the lowest electrical conductivity? a. HCl b. K2SO4 c. NaCl d. HF e. Ba(NO3)2

d. HF -All the given substances are strong electrolytes (strong acid: HCl, soluble salts: K2SO4, NaCl, and Ba(NO3)2) EXCEPT for HF (weak acid) which is a weak electrolyte. Thus, HF would have the lowest amount of ions in solution and would exhibit the lowest electrical conductivity

26. Which of the following would be strongly attracted to magnetic fields? a. He b. H2 c. Zn d. O2 e. N2

d. O2 -A paramagnetic atom or molecule has at least one unpaired electron which will result in a tiny magnetic field that can interact with a larger magnetic field. -If all of the electrons are paired within their orbitals, then the molecule is diamagnetic and will not interact with magnetic fields -N2 is diamagnetic and O2 is paramagnetic

7. Which of the following molecules has a net nonzero dipole moment? a. CCl4 b. CO2 c. PCl5 d. PF3 e. BF3

d. PF3

4. Which of the following gases will have the greatest density at STP? a. Cl2 b. N2O c. Ar d. SO3 e. CO2

d. SO3 density = P(MM)/RT -Since all the gases are at the same T and P, the density is only dependent on the molar mass

25. Which of the following is the most likely formed compound between atoms X and Y? Atom X: 1s22s22p63s2 Atom Y: 1s22s22p4 a. X3Y b. X2Y c. X2Y3 d. XY e. XY2

d. XY -Using the periodic table, we can find that atom X is magnesium and atom Y is oxygen. These atoms are most often found in with a +2 and -2 charge in compounds, respectively. Therefore, we only need one of each atom to form the likely compound MgO

25. An isotope of oxygen will result in which of the following? a. a change in the valence electrons b. a change in the reactivity of oxygen c. a change in the atomic number d. a change in the atomic mass e. a change in the electronegativity

d. a change in the atomic mass -An isotope is an element that has a differing number of neutrons. The charge of an isotope does not change, only the atomic weight of the element changes, [A]-change in the valence electrons would result in an ion, not an isotope [C]- atomic number never changes

27. An unknown element X is solid at room temperature, reacts with chlorine to form XCl2, and oxides of X form a basic solution in water. X is most likely a(n) a. alkali metal b. halogen c. noble gas d. alkaline earth metal e. nonmetal

d. alkaline earth metal -Alkaline earth metals (Group IIA: Be, Mg, Ca, Sr, Ba, Ra) are solid at room temperature and have a general valence electron configuration of ns2. They tend to lose their two outer s electrons and form +2 ions. -Their oxides, like all metal oxides, form basic solutions in water

18. An exothermic reaction at equilibrium will proceed at a: a. slower rate while its Kc increases at higher temperatures b. slower rate while its Kc decreases at higher temperatures c. faster rate while its Kc increases at higher temperatures d. faster rate while its Kc decreases at higher temperatures

d. faster rate while its Kc decreases at higher temperatures A + B → C + D + heat Kc = [C][D]/[A][B] -Le Chatlier's principle, increasing the temperature would increase the heat, so therefore the reaction would shift to the left (towards the reactants, [A] and [B]). -If the concentrations of [A] and [B] increase, then the overall Kc of the reaction would decrease at higher temperatures -increasing the temperature will increase the rates of both the forward and the reverse reactions, so the reaction will proceed at a faster rate.

8. A sample of 50 g liquid acetone, CH3COCH3 and 50 g liquid HCl contain all of the following intermolecular forces EXCEPT one. Which one is the EXCEPTION? a. dispersion b. dipole-dipole c. Van der Waals d. hydrogen bonding e. induced dipole

d. hydrogen bonding

23. Two half-reactions of silver(I) solution are set up in an galvanic cell with differing concentrations. In this concentration cell, which of the following is driving the current? a. voltage from an external battery source b. attraction of electrons to the higher concentration of Ag+ ions c. deposition of balancing ions from the salt bridge into the half reactions d. tendency for the two differing concentrations to equalize due to entropy e. deposition of electrons and ions onto the anode

d. tendency for the two differing concentrations to equalize due to entropy -In a normal galvanic cell, two different half reactions are used. In this case, the same half reaction is used, silver(I), but the two half reactions have different concentrations. As we know from general chemistry principles, two unequal concentrations will tend to equalize due to entropy

13. Which statement explains why HI is a stronger acid than HBr, which is a stronger acid than HCl? a. iodine is the smallest element out of the 3 and thus creates the strongest acid b. the acid strength is increased as electronegativity increases in the binding element c. the number of available electrons in iodine for binding is greater than that of Cl or Br d. the bond strength between H-I is lower than that of H-Cl or H-Br e. acid dissociation decreases as the shielding effect increases

d. the bond strength between H-I is lower than that of H-Cl or H-Br [A]-iodine is the largest element out of the three [B]-acid strength is decreased as electronegativity increases in the halogens [C]-all of the halides have 7 available valence electrons [E]-the shielding effect increases as you go down a column, and the acid strength also increases in the halides

25. The differentiating electron is the last electron to be filled within an atom. The differentiating electron of an atom is given by the quantum numbers n = 3, l = 2, ml = 0, and mms = -½. Assume negative values first for the magnetic quantum number, ml, and positive values first for the spin quantum number, ms. All of the following statements are true EXCEPT one. Which one is the EXCEPTION? a. the element is a metal b. the atomic number of the element is 28 c. the differentiating electron is a d electron d. the element has 8 valence electrons e. the atom is paramagnetic

d. the element has 8 valence electrons -when n=3 and l=2, the electron is in a 3d subshell -A "d" subshell has 5 orbitals, each assigned an ml value of -2, -1, 0, +1, and +2. An ml=0 value indicates that the electron is on the third orbital. -An ms= -1/2 indicates that this electron is paired -[Ni] = (1s^2)(2s^2)(2p^6)(3s^2)(3p^6)(4s^2)(3d^8) -Ni is a metal with atomic number 28 and is paramagnetic (with two unpaired electrons) -If we are looking at the d block, then the d electrons and the highest s electrons are the valence electrons. -For Ni, the two 4s electrons and the eight 3d electrons are the valence electrons, therefore Ni has 10 valence electrons.

18. All of the following statements are true of entropy EXCEPT for one. Which one is the EXCEPTION? a. entropy will increase at temperature increases b. entropy is a measure of the disorder in a system c. a spontaneous reaction can produce an increase in entropy d. the entropy of a forward reaction is the same as the entropy of the reverse reaction e. the entropy of a gas vessel increases as volume increases

d. the entropy of a forward reaction is the same as the entropy of the reverse reaction

3. The combustion of hydrogen gas occurs according to the equation below. This observation is supported by which of the following? O2(g) + 2H2(g) → 2H2O(g) a. Dalton's law of partial pressure b. the zeroth law of thermodynamics c. Dalton's atomic theory d. the law of conservation of mass e. the law of conservation of charge

d. the law of conservation of mass -During a chemical reaction, the mass of the reactants is the same as the mass of the products (no creation of atoms, no destruction of atoms) [B]-zeroth law of thermodynamics tells us that if two systems are each in thermal equilibrium with a third system, then all systems are in thermal equilibrium with each other. [C]-Dalton's atomic theory states that all matter is composed of atoms, which can combine in ratios to form compounds, and that atoms differ based on size/mass.

22. For the redox conversion CrO4^2- → CrO2^-, all of the following statements are correct EXCEPT for one. Which one is the EXCEPTION? a. when CrO4^2- → CrO2^-, Cr is reduced b. there are 3 electrons involved per mole of CrO4^2- c. the oxidation number of Cr in CrO4^2- is +6 d. this represents an oxidation half reaction e. Cr in CrO2^- has a positive oxidation number

d. this represents an oxidation half reaction

30. Why should one always slowly add acid to a beaker of water rather than water to a beaker of acid? a. to ensure the acid doesn't react with impurities in beaker b. to prevent the water from sinking beneath the acid and remain unmixed c. to maximize the ionization of the acid being diluted d. to ensure there is enough water to absorb the heat released

d. to ensure there is enough water to absorb the heat released

30. Which separation technique is based on differences in the particle size of the substances being separated? a. fractional distillation b. liquid-liquid solvent extraction c. open dish evaporation d. vacuum filtration e. thin layer chromatography

d. vacuum filtration [A]-Fractional distillation is a method to separate the components of a solution. The solution is gradually heated up, and the substance with the lower boiling point will boil first and exit the solution as a gas [B]-Solvent extraction is when two immiscible liquids are shaken in a container that contains solutes. The solutes will associate with the solvent they are attracted to and will separate based on density

10. If KCl is added to distilled water, which of the following properties would decrease in value? a. molality of the solution b. molarity of the solution c. boiling point of the solution d. vapor pressure of the solution e. boiling point of the pure solvent

d. vapor pressure of the solution -The presence of a solute will lower the vapor pressure of the solution, compared to the vapor pressure of the pure solvent, via colligative properties. Because the solute is nonvolatile, the vapor above the solution will still be just solvent molecules, but the vapor pressure of the solution will be lower with the solute added.

8. Which of the following correctly describes an allotrope?

different structural forms of the same element

12. What is the pH of a solution prepared by adding 20 mL of 0.02 M NaOH(aq) to 20 mL of water? a. 2 b. 3 c. 10 d. 11 e. 12

e. 12 NaOH is a strong base and fully dissociates into Na+ and OH-. To find the new concentration of NaOH after we dilute it with water, we use: M1V1 = M2V2 (0.02)(0.02) = M2(0.04) 0.01 = M2 -we use 0.04L for the second volume, NOT 0.02L because we are adding 0.02L of NaOH(aq) to 0.02L of water for a total of 0.04L -since NaOH is a strong base, we can conclude [OH-]= 0.01 M pOH = -log([OH-]) pOH = -log(0.01) pOH = 2 pH + pOH = 14 pH = 14 - pOH pH = 14 - 2 = 12

4. A closed 1L chamber of gas undergoes the following reaction at 400 °C and 20,000 kPa: 2H2(g)+ O2(g) --> 2H2O(g) Assuming 2 mols of hydrogen gas, 1 mol of oxygen gas, a constant temperature, and no product at start, what is the resulting pressure after the reaction occurs? a. 9,333 kPa b. 9,666 kPa c. 11,000 kPa d. 11,666 kPa e. 13,333 kPa

e. 13,333 kPa P1V1/n1T1= P2V2/n2T2 P1= 20,000 n1= 3 mol n2= 2 mol T1 and T2= 400+273= 673k V1 and V2= 1L

23. What is the maximum number of electrons that can be held in the 4th energy level? a. 2 b. 8 c. 10 d. 14 e. 32

e. 32 -The fourth energy level has four orbitals, s, p, d, and f s = 2 e- p = 6 e- d = 10 e- f = 14 e- (2 + 6 + 10 + 14) = 32 electrons

7. All of the following are strong electrolytes EXCEPT for one. Which one is the EXCEPTION? a. HNO3 b. H2SO4 c. NaCl d. KOH e. CO2

e. CO2 -"one of the following is not alike". you can guess the answer is [E] if you recognize that it isn't an ionic compound or something that breaks apart and dissolves in water -NaCl and KOH both will dissolve into two ions

21. Which of the following can act as both an oxidizing agent and a reducing agent? a. NO3^-1 b. Co c. F^-1 d. MnO4^-1 e. H2O2

e. H2O2 -the oxygen in H2O2 is at a -1 oxidation state. The oxygen can become further reduced into a -2 oxidation state, such as in H2O, or it can become oxidized to a 0 oxidation state in O2 [A]: NO3^-1 N = +5 [B]: Co = 0 [C]: F^-1 F = -1 [D]: MnO4^-1 Mn = +7

27. Which element has the smallest first ionization energy? a. N b. C c. B d. Be e. Li

e. Li -ionization energy increases up and to right on the periodic table

6. Which statement best explains why bromine is a liquid and iodine is a solid at STP? a. at a given temperature, bromine has a higher vapor pressure than iodine b. smaller atoms tend to be more polarizable and exhibit greater intermolecular forces c. the dipole-dipole interactions in iodine are stronger than in bromine d. elements in the same column tend to have different chemical properties e. London dispersion forces increase as molar mass increases

e. London dispersion forces increase as molar mass increases [A]-Vapor pressures are a result of intermolecular forces and internal energy, not a cause of physical states of matter. [B]-larger molecules tend to be more polarizable and exhibit greater intermolecular forces. [C]-both iodine (I2) and bromine (Br2) are nonpolar and would exhibit London dispersion forces, not dipole-dipole [D]-elements in the same column tend to have similar chemical properties

19. All of the following will affect the rate of an irreversible chemical reaction EXCEPT one. Which one is this EXCEPTION? a. pressure b. concentration of reactants c. presence of a catalyst d. surface area of reactant solid e. concentration of products

e. concentration of products -Increasing the pressure will make more molecules hit each other, resulting in a change in the rate of the reaction -Increasing the concentration of the reactants will affect the frequency of molecular collisions -A catalyst will lower the activation energy required and speed up a reaction -More surface area of a reactant will give more opportunities for molecules to strike to cause a reaction

8. All of these are colligative properties EXCEPT for one. Which one is the EXCEPTION? a. change in vapor pressure b. freezing point depression c. osmotic pressure d. boiling point elevation e. density

e. density -Colligative properties are based on the number of solute and solvent particles in a solution, NOT the identity of the particles themselves. -Molality is used for vapor pressure, boiling point elevation, and freezing point depression. -Molarity is used for osmotic pressure. Thus, dissolved species can reduce vapor pressure, decrease freezing point and increase boiling point, and also change the osmotic pressure.

26. Calcium has a larger atomic radius than magnesium because of the: a. difference in the number of neutrons in their nucleus b. increase in ionization energy c. lanthanide contraction d. increase in effective nuclear charge e. increase in electron shielding

e. increase in electron shielding -many of the periodic trends, including atomic size, can be derived from the two concepts of effective nuclear charge and electron shielding 1. The increasing effective nuclear charge is responsible for the decrease in atomic size across the periodic table from left to right 2. The increasing electron shielding effect is responsible for the increase in atomic size moving down groups

19. A reaction has a rate law of: R = k[A]3[B] Which of the following would happen to the rate, if the concentration of A is doubled and the concentration of B is held constant? a. it would stay the same b. it would increase by a factor of 2 c. it would increase by a factor of 4 d. it would increase by a factor of 6 e. it would increase by a factor of 8

e. it would increase by a factor of 8 -For substance A, which is cubed, doubling the concentration would result in a rate increase by a factor of 2^3 or 8

7. A student planning an experiment reads that CCl4 melts at -22.9 °C, while NaCl melts at 801 °C. All of the following contribute to this observation EXCEPT one. Which one is the EXCEPTION? a. the intermolecular forces in CCl4 are weaker b. CCl4 lacks lattice energy c. the intramolecular forces in CCl4 are weaker d. NaCl has a larger electronegativity difference between atoms e. melting CCl4 breaks intramolecular bonds

e. melting CCl4 breaks intramolecular bonds -there is less of an electronegativity difference between the atoms involved in a covalent bond, covalent substances (CCl4) have weaker intramolecular and intermolecular bonds than ionic substances (NaCl). When a substance melts, it undergoes a phase change from solid to liquid.

27. Three elements in the same period are listed in order of decreasing atomic radius. Which of the following is an appropriate explanation for the non-metal in the list having the smallest atomic radius? a. non-metals have an expanded octet b. non-metals have higher electronegativities c. non-metals have fewer electron orbitals d. non-metals have higher ionization energies e. non-metals have a higher effective nuclear charge

e. non-metals have a higher effective nuclear charge 1. Atomic radius increases from top to bottom of a family (column) -the increase in electrons causes an increase in orbitals to house them and an increase in atomic radius 2. Atomic radius decreases across the period from left to right -Effective Nuclear Charge= for non-metals, their negative electrons are more attracted to the positive nucleus resulting in a compacting of the atomic size

15. The conjugate base of which of the following acids will have the highest Kb? a. pKa = 3.46 b. pKa = 3.74 c. pKa = 4.72 d. pKa = 8.64 e. pKa = 9.29

e. pKa = 9.29 ↑ pKa = ↓ Ka = weaker acid = strong conjugate base ↑ Kb = stronger base

8. A solid compound that has a vapor pressure higher than atmospheric pressure will most likely be able to do which of the following? a. melt b. freeze c. condense d. vaporize e. sublime

e. sublime -If a solid compound has a high internal vapor pressure relative to the external environment, the compound will be able to sublime as the particles will be able to readily turn into vapor without first transitioning into the liquid state

11. An aqueous solution of potassium chloride is heated from 20C to 60C (no vaporization occurred). Which of the following occurs while the solution is being heated? a. the mole fraction of solute decreases b. the mole fraction of solvent increases c. the density of solution is constant d. the molarity of solution is constant e. the molality of solution is constant

e. the molality of solution is constant -the volume of the solution is changing due to the added heat (thermal expansion) -mole fractions are unaffected by volume changes -density of the solution does NOT stay constant, because the volume changes -molarity of the solution does NOT stay constant, because it depends on volume, or liters -molality of a solution only depends on the mass of the solvent, in kg, which does not change with temperature

23. Which molecular shape is characterized by having 3 bonding domains and 1 non-bonding domain? a. tetrahedral b. bent c. seesaw d. trigonal planar e. trigonal pyramidal

e. trigonal pyramidal -a compound with 3 bonding domains means the central element is bound to 3 other elements. In addition, there is 1 non-bonding domain, which means that there are a pair of electrons that are not engaged in a bond. The total (non-bonding and bonding) domains is 4

isenthalpic process

enthalpy remains constant [∆H = 0]

isentropic process

entropy remains constant [∆S = 0]

adiabatic process

heat energy remains constant [∆q = 0]

18. Calcium carbonate decomposes according to the equation given below. All of the following statements about this reaction are true EXCEPT one. Which one is the EXCEPTION? CaCO3(s) ⇄ CaO(s) + CO2(g) Keq = 20

if the concentration of the reactants are doubled, Keq doubles -liquids and solids are NEVER included in the expression for Keq for this given equation, the only reactant is a solid. Therefore, changing the concentration of the reactants would have no effect on Keq -changing the temperature of the reaction will change the Keq -changing the reaction itself by reversing the reaction or by adding coefficients to every substance in the reaction will change the Keq -When a reaction is reversed, its Keq value is the reciprocal of the Keq of the forward reaction

30. A student finds that the average number of cells over 10 samples is 2130, with a standard deviation of ±6. However, the professor determines the true average number of cells is 2850. The student's measurement was:

inaccurate and precise -Accuracy= that the measurement taken is close to the true value -Precise= that the measurement produces consistent results -the student's measurements were precise, since we can see the standard deviation is low compared to the total number of cells, meaning the variation of each measurement was low, but the measurement was inaccurate because it differed greatly from the true value

11. Which of the following changes will result in the greatest increase in solubility of O2 gas in water?

increase pressure of the gas and decrease temperature of the solution -in a system where gas exists above a liquid, the solubility of the gas is affected by both pressure of the gas and the temperature of the solution ↑ pressure of the gas = ↑ gas solubility ↑ temperature of the solution = ↓ gas solubility

N2(g) + 3H2(g) ⇌ 2NH3(g) ; ΔH° = -92.22 kJ•mol-1 Which change would decrease the Keq in the above reaction?

increasing the temperature -the only thing that can change the Keq is changing the temperature

13. When titrating the monoprotic acid HCl with NaOH of a known concentration, which of the following must be equal at the equivalence point?

initial moles of HCl are equal to the moles of OH- added

13. 50 mL of each of the following solutions is added to 50 mL of 0.1 M acetic acid. The total volume of the resulting solution is 100 mL. Examining the resulting solutions, which would have the lowest pH and which the highest pH? 0.1 M H2CO3 0.1 M HBr 0.1 M KOH

lowest pH: HBr, highest pH: KOH HBr = strong acid (low pH) KOH = strong base (high pH) H2CO3 = weak acid

14. Barium hydroxide dissociates into ions, as shown in the reaction below. If there is 0.0005M barium hydroxide, what is the pH of the solution? Ba(OH)2(aq) →Ba^2+(aq) + 2OH^-(aq)

pH = 11 -for every 1 mol Ba(OH)2 we produce 2 mol OH^- [0.0005 mol Ba(OH)2/ 1L] * [2 mol OH^-/ 1 mol Ba(OH)2] = 0.001 M OH^- pOH = -log[OH-] = -log(1 × 10^-3) = 3 14 = pH + pOH pH = 14 - pOH pH = 14 - 3 pH = 11

isobaric process

pressure remains constant [∆P = 0]

5. Gas A has a molar mass of 4 g/mol and gas B has a molar mass of 36 g/mol. If the gases are at the same temperature, what will be the rate of effusion of gas B relative to gas A?

rate of gas B= 1/3 * rate of gas A Graham's Law: rate of effusion rate1/rate2 = √MM2/MM1 rateA/rateB = √36/4 = √9 = 3 -we would expect gas A (4 g/mol) to effuse faster than gas B (36 g/mol)

19. A particular reaction has a rate constant of 8.5 × 108 M-1s-1. What is the overall order of this reaction?

second order -1 + -1 = 2

9. The preferred colligative property for measuring the molecular weight of polymers is osmotic pressure. Which of the following best explains why osmotic pressure is preferred to measure the molecular weight of polymers?

small changes are easier to measure in osmotic pressure than in other colligative properties -Technically, you could use any colligative property (freezing point, boiling point) to measure the molecular weight of a compound. But osmotic pressure is preferred for polymers (5,000+ g/mol) because polymers are usually very insoluble. -If osmotic pressure changes slightly, the height of the solution column will raise or lower (depending on the change). And this height change can be easily observed

9. In a closed system at equilibrium, the partial pressure of undissolved oxygen is tripled. How will the new oxygen solubility (solubility2) compare to the original oxygen solubility (solubility1)?

solubility2 = 3*(solubility1) Henry's law tells us that the solubility of a gas in liquid is directly proportional to the partial pressure of the gas above the solution. C = kP where C = concentration of gas in solution, k is a proportionality constant, and P is the partial pressure of the gas above the solution. -if the partial pressure is tripled, the solubility will triple.

isothermal process

temperature remains constant [∆T = 0]

25. How would the bond energy and bond length of a carbon-carbon bond in C2H6 compare to C2H4?

the bond energy for C2H4 would be greater, but the bond length would be shorter

19. Consider a hypothetical reaction 2A + B → 3C, with a rate that is second order with respect to B and zero order with respect to A. How would doubling the concentration of A and increasing the concentration of B by a factor of three affect the rate of this reaction?

the rate will increase by a factor of nine Rate = k[B]^2 -[A] is zero order and won't appear in the rate law -increasing the concentration of B by a factor of three will increase the rate by a factor of nine (3^2 = 9)

5. Which is the best definition of a boiling point?

the temperature at which the vapor pressure of the liquid equals the surrounding pressure

12. A chemist is performing a distillation experiment of a miscible solution containing equal parts liquid X and liquid Y. The vapor pressure of pure liquid X is 400 mm Hg, and the vapor pressure of pure liquid Y is 200 mm Hg. Based on this information, which of the following findings would the chemist likely observe?

the vapor above the solution of X and Y contains more molecules of gas X than gas Y ↑ Vapor pressure = ↓ Lower boiling point ↑ Vapor pressure = ↑ Volatility -Liquid X has a higher vapor pressure (400 mm Hg) than liquid Y (200 mm Hg). Therefore X is more volatile, will boil at a lower temperature, and will contribute more to the vapor pressure above the solution than liquid Y

16. An equilibrium mixture for the reaction A⇌B has a Kc = 0.40 at 350K. Which of the following statements is true regarding a A⇌B mixture with a Qc = 20 at 350K?

to reach equilibrium, the concentration of A will increase while the concentration of B will decrease [Q<K] = more reactants than products, shift right [Q>K] = more products than reactants, shift left [Q=0] = since Q is less than K, shift right [Q=K] = system is at equilibrium

19. 2A2B → 4A + 2B k = 2.0 x 10-4 M·s-1 What is the reaction order of the above equation?

zero order -We see that the units are M·s-1, or M1·s-1. To find the reaction order, we just add up all exponents and take the absolute value. In this case, 1 + (-1) = 0, so this is a zero-order reaction

1. 2 H2 (g) + O2 (g) → 2 H2O (g) (ΔH) = -242 kJ/mol 2. H2O (g) → H2O (l) (ΔH) = -286 kJ/mol According to Hess' Law, what is the total enthalpy for the equation: 2 H2O (l) → 2 H2 (g) + O2 (g)

ΔHrxn = (-1)(-242) + (-2)(-286) = 242 kJ (from step 1) + 572 kJ (from step 2) = 814 kJ/mol 1. step 1 must be reversed: 2 H2O (g) → 2 H2 (g) + O2 (g) (ΔH) = -1(-242 kJ/mol) 2. step 2 must be reversed and doubled: 2 H2O (l) → 2 H2O (g) (ΔH) = -2(-286 kJ/mol) -in the final equation, H2O (l) is on the left side of the arrow, but in step 2, the compound is on the right side of the arrow. -there is no H2O (g) in the final equation. Therefore, it has to be eliminated when the two equations are added together. There are 2 H2O (g) in step 1, but only 1 H2O (g) in step 2 -when reversing an equation: (ΔH) is multiplied by -1 -when doubling an equation: the coefficients as well as the (ΔH) are multiplied by 2

17. The second law of thermodynamics deals with the spontaneity of the change of matter in terms of entropy (S). A change is spontaneous if

ΔS(surrounding) + ΔS(system) > 0 -Every spontaneous change is accompanied by an increase in the entropy of the universe (system and surroundings)

17. What will be the effect on the solubility of BaF2 if the following changes are made: increasing temperature; adding NaF; adding HCl?

↑ temperature = ↑ solubility adding NaF (common ion) = ↓ solubility adding HCl (more acidic) = ↑ solubility


Kaugnay na mga set ng pag-aaral

Mastering A&P Intro into the Nervous System

View Set

Online Lecture - Chest Tube Management

View Set